You are on page 1of 81

Page 1 of 81

1. A 14-year-old girl with seizure disorder is admitted to the Shadow General pediatric floor.
During her hospitalization she is diagnosed with Bells palsy. What would you expect to find
on physical exam?
A.
B.
C.
D.

Increased peripheral deep tendon reflexes


Decreased peripheral deep tendon reflexes
One-sided facial paralysis
One-sided body paralysis

2. A 72-year-old with a history of Parkinsons is admitted to Shadow General Hospital for


shoulder surgery. What would you expect to find on the neurological exam?
A.
B.
C.
D.

Tremor at rest
Flaccid arm muscles
Flaccid leg muscles
Tremor with activity

3. Which of the following could be used to assess abstract thought?


A.
Asking Tina to explain an idiom like, A rolling stone gathers no moss.
B.
Drawing the number 3 in Tinas hand and asking her to identify the number.
C.
Placing a paperclip in Tinas hand and asking her to identify the object.
D.
Asking Tina to explain a common phrase such as, It is a beautiful day.
A: A person exhibiting abstract thinking will be able to identify the intended connotation of a
phrase. A concrete thinker will only be able to describe the literal meaning. Abstract thinking can
also be assessed by asking about similarities between objects.
4. You examine cerebellar function by examining:
A
Coordination
B
Reflexes
C
Flexibility
D
Strength
A: The cerebellum is responsible for creating fluency and accuracy in voluntary motions. You
can test cerebellar function by assessing a persons coordination and purpose in movements
while ambulating and while performing the finger-to-finger, finger-to-nose, heel-to-shin, rapid
alternating movements, and Romberg tests.
5. To determine Tinas level of consciousness, you would assess her orientation to
A.
Place, situation, and mediation list
B.
Person, place, and time
C.
Time, language, and family history
D.
Person, place, and recent memory
B: The first sign of deterioration of a patients condition is usually changes in level of consciousness.

Page 2 of 81
6. If during an exam, Tina told you that her feet occasionally felt numb, what assessment
would you do?
Because Tina is at risk for diabetic neuropathy, it is important to assess bilateral
sensations starting at her feet and proceeding up the legs. Sensation tests should include sharp
and dull touch, light touch, vibration, temperature, point location, and positioning of joints. Superficial and deep tendon reflexes of the feet, ankles, and knees should also be assessed to
determine the extent of her neuropathy.
7. Imagine that the documentation in the Electronic Medical Record (EMR) described Tina as
confused yesterday evening. However, you had assessed orientation and basic level of consciousness to find her alert and oriented to person, place and time. Would it be necessary to
assess Tinas mental status further?
Usually mental status can be informally assessed during a patient interview.
However, due to Tinas recent fluctuation in cognition, a formal mental status exam should be
performed to ensure that she is lucid in all domains.
8. Identify the components (domains) of a comprehensive mental status exam.
Comprehensive mental status exams should assess a patients orientation, attention, abstract thinking, comprehension, judgment, memory, vocabulary, and speech. Mental status assessment is usually performed informally and continuously throughout the physical exam.
When a problem is suspected, the clinician may choose to perform a complete formal mental
status exam.
9. A 3-year-old child with Downs syndrome is in the cardiac ICU of Shadow General Hospital
following open heart surgery. Which of the following musculoskeletal physical exam findings
would you expect to find on this child?
A
Fixed joints
B
Lordosis
C
Hyperextensible joints
D
Scoliosis

10. An 86-year-old woman has been admitted to the Shadow General surgical floor after a knee
replacement. She has a history of degenerative joint disease (DJD). What physical exam findings would you expect to find?
A
Joint deformity
B
Shuffling gait
C
Postural instability
D
Humpback

11. If Tina had displayed limited abduction of the left shoulder, it would mean that:
A
She has limited movement forward
B
She has limited movement in a circular motion

Page 3 of 81
C
D

She has limited movement toward her body


She has limited movement away from her body

D: Abduction is defined as a motion in which a body part is moved away from the bodys midline.
12. Suppose you noted that Tinas left knee was swollen. Which of the following tests should be
done?
A
Balloon test
B
Fluid wave
C
Bulge sign
D
Tinel's sign

C: The bulge sign is a test for excess fluid accumulation in the knee. It is performed by rubbing
the knee capsule in upward strokes to displace fluid and observing for the creation of a bulge
when the fluid returns to its original position.
13.Imagine that Tina was unable to move her right ankle. You notice that her muscles contract
as she attempts to dorsiflex her ankle, but no movement occurs. How would you grade her ankle strength?
A
1/5
B
2/5
C
4/5
D
3/5

A: According to the Oxford Scale, a muscle group that has no mobility but displays visible muscle contraction is graded as a 1/5 strength.
14. The group of axillary lymph nodes that drains the other three groups of nodes is the:
A
anterior nodes.
B
central nodes
C
lateral nodes.
D
posterior nodes.

B The central axillary nodes receive lymph from the other three groups of nodes (i.e., anterior
[pectoral], posterior [subscapular], and lateral).

15.An older adult female is having an annual mammogram. Before the mammogram, the nurse
does a breast examination. Expected normal findings would include:
A
palpable, firm, stringy lactiferous ducts.
B
increased glandularity.
C
yellow colostrum expressed from the nipple.
D
a unilateral venous pattern.

Page 4 of 81
A An older adults (eighth to ninth decades) lactiferous ducts are more palpable and feel firm
and stringy because of fibrosis and calcification. The older adults glandular breast tissue atrophies. Yellow colostrums may be expressed from the nipple of a pregnant female after the first
trimester. A venous pattern is prominent over the skin surface of pregnant women.
16. The number of lobes within the breast ranges between:
A
10 and 20.
B
15 and 20.
C
20 and 25.
D
20 and 40.

B The glandular breast tissue contains between 15 and 20 lobes radiating from the nipple.
17. Gynecomastia occurs with:
A
Addison disease.
B
hypothyroidism.
C
calcium channel blockers.
D
liver cirrhosis.

D Gynecomastia occurs in liver cirrhosis because the liver is unable to metabolize estrogens.
Gynecomastia may occur in hyperthyroidism. Gynecomastia may occur in Cushing syndrome
not Addison disease. Gynecomastia may occur as an adverse effect of certain medications
(e.g., metronidazole, isoniazid, digoxin, ACE inhibitors, diazepam, and tricyclic antidepressants)
18. The relative proportion of glandular, fibrous, and adipose breast tissue depends on
A
environmental factors.
B
genetics.
C
sex.
D
nutritional state.

D The relative proportion of glandular, fibrous, and fatty tissue varies depending on age, cycle,
pregnancy, lactation, and general nutritional state.
19. A known risk factor for breast cancer includes:
A
early menarche or late menopause.
B
low breast tissue density.
C
breastfeeding an infant for more than 6 months.
D
low-fat, low cholesterol diet.

A. A risk factor for breast cancer is early menarche (less than 12 years) or late menopause (after 55 years). A risk factor for breast cancer is high breast tissue density. A risk factor for breast
cancer is never breastfeeding a child. A risk factor for breast cancer is a high-fat diet.
20. A normal common breast variation is:
A
enlarged axillary lymph nodes.
B
a supernumerary breast.
C
a supernumerary nipple.

Page 5 of 81
D

fixation of the breast.

C. A supernumerary nipple is a normal and common variation. An extra nipple on the thorax or
abdomen is a congenital finding. It is usually 5 to 6 cm below the breast near the midline and
has no glandular tissue. Fixation of the breast indicates invasive cancer that fixes the breast to
the underlying pectoral muscles. A supernumerary breast is rare; there is additional glandular
tissue present. Enlarged axillary lymph nodes indicate a local infection of the breast, arm, or
hand, or breast cancer metastases.
21. An expected postmenopausal breast change includes:
A
increased glandular tissue.
B
decreased fibrous connective tissue.
C
increased fatty tissue.
D
decrease in breast size.

D. After menopause expected breast changes include a decrease in breast size. After menopause expected breast changes include fat tissue atrophy. After menopause expected breast
changes include increased fibrous connective tissue. After menopause expected breast changes include breast glandular tissue atrophy.

22. The most common site of cancerous breast tumors is in the:


A
upper inner quadrant.
B
upper outer quadrant.
C
lower inner quadrant.
D
lower outer quadrant.

B. The most common site of breast cancer is in the upper outer quadrant.
23. Breast development in the adolescent female usually:
A
occurs after the beginning of menstruation.
B
precedes menstruation by about 2 years.
C
begins between 12 and 13 years of age.
D
takes an average of 5 years.

B. Breast development usually precedes menstruation by about 2 years. Breast development usually begins between 8 and 9 years for African American girls and by 10 years for
white girls Full breast development from Tanner stage 2 to 5 takes an average of 3 years, although the range is 1.5 to 6 years.
24. The reservoirs for storing milk in the breasts are:
a. lobules
b. alveoli
c. Montgomery's glands
d. lactiferous sinuses

Page 6 of 81
D Lactiferous sinuses
25. During a visit for a school physical, the 13-year-old girl being examined questions the
asymmetry of her breasts. The best response is:
a. "One breast may grow faster than the other during development."
b. "I will give you a referral for a mammogram."
c. "You will probably have fibrocystic disease when you are older."
d. "This may be an indication of hormonal imbalance. We will check again in 6-months.

A "One breast may grow faster than the other during development.
26. When teaching the breast self-exam, you would inform the woman that the best time to conduct breast self-examination is:
a.
at the onset of the menstrual period.
b.
on the 14th day of the menstrual cycle.
c.
on the 4th to 7th day of the cycle.
d.
just before the menstrual period.

C On the 4th to 7th day of the cycle


27. This is the first visit for a woman, age 38. The practitioner instructs her that a baseline
mammogram is recommended for women between the ages of 35 and 39 and that the clinical
examination schedule would be based on age. The recommendation for women 40 to 49 is:
a. every year
b. every 2 years
c. twice a year
d. only the baseline exam is needed unless the woman has symptoms.

A. Every year
28. The examiner is going to inspect the breasts for retraction. The best position for this part of
the examination is:
a. lying supine with arms at the sides.
b. leaning forward with hands outstretched.
c. sitting with hand pushing onto hips.
d. one arm at the side, the other arm elevated.
C. sitting with hand pushing onto hips.
29. A bimanual technique may be the preferred approach for a woman:
a. who is pregnant
b. who is having the first breast examination by a health care provider.
c. with pendulous breasts
d. who has felt a change in the breast during self examination.

C. with pendulous breasts

Page 7 of 81

30. During the examination of a 70-year-old man, you note gynecomastia. You would:
a. refer for a biopsy
b. refer for a mammogram
c. review the medications for drugs that have gynecomastia as a side effect.
d. proceed with the exam. This is a normal part of the aging process.

C. review the medications for drugs that have gynecomastia as a side effect.
31. During a breast examination, you detect a mass. Identify the description that is most consistent with cancer rather than benign breast disease.
a. round, firm, well demarcated
b. irregular, poorly defined, fixed
c. rubbery, mobile, tender
d. lobular, clear margins, negative skin retraction.
B. Irregular, poorly defined, fixed
32. During the examination of the breasts of a pregnant woman, you would expect to find:
a. peau d' orange
b. nipple retraction.
c. a unilateral, obvious venous pattern
d. a blue vascular pattern over both breasts.

D. A blue vascular pattern over both breasts.


33. Which of the following women should not be referred to a physician for further evaluation?
a. a 26-year-old with multiple nodules palpated in each breast.
b. a 48-year-old who has a 6-month history of reddened and sore left nipple and areolar
area.
c. a 25-year-old with asymmetrical breasts and inversion of nipples since adolescence.
d. a 64-year-old with ulcerated area at tip of right nipple, no masses, tenderness, or
lymph nodes palpated.

C. a 25-year-old with asymmetrical breasts and inversion of nipples since adolescence.


34. Breast asymmetry:
a. increases with age and parity.
b. may be normal.
c. indicates neoplasm.
d. is accompanied by enlarged axillary lymph nodes.

B. may be normal
35. Any lump found in the breast should be referred for further evaluation . A benign lesion will
usually have 3 of the following characteristics. Which one is characteristic of a malignant lesion?
a. soft

Page 8 of 81
b. well-defined margins
c. freely movable
d. irregular shape

d. irregular shape

36. What is Gynecomastia? (Short answer)

Overgrowth of glandular tissue


37. Which is the first physical change associated with puberty in girls?
a. areolar elevation
b. breast bud development
c. height spurt
d. pubic hair development
e. menarche

B. Breast bud development


38. What lifestyle factors increase a woman's risk for breast cancer? (Select all that apply)
a. oral contraceptive use
b. never breastfed
c. alcohol intake of >1 drink daily
d. obesity, physical inactivity
ABCD

*The next set of questions are short answer*


39. How does a normal breast feel?

ANS: Firm, smooth, & elastic


40. How does breast tissue feel after pregnancy?

ANS: Softer & loser


41. What postpartum day does colostrum change to milk production?

ANS: 3rd day

Page 9 of 81

42. What area has the greatest amount of glandular tissue and is where cancer is most likely to
be found?

ANS: Tail of Spence


43. What are the 4 different positions for breast exam?

ANS: Arms relaxed at side, hands on hips, arms raised above head, bending forward

44. What are the 4 different palpation patterns for breast exam?

ANS: Glyde, Vertical, wedge, spiral

45. What may be present in the lower curve of the breast below the nipple?

ANS: Small area of firm tissue

46. What causes galactorrhea?

ANS: Nursing, breast stimulation, horomones

47. What is peau dorange?

ANS: Orange peeled skin from clogged duct


***MULTIPLE CHOICE***

48. Increased tactile fremitus would be evident in an individual who has which of the following
conditions?
A
Emphysema

Page 10 of 81
B
C
D

Pneumonia
Crepitus
Pneumothorax

B. Fremitus is a palpable vibration. Increased fremitus occurs with compression or consolidation


of lung tissue (e.g., lobar pneumonia). Decreased fremitus occurs when anything obstructs
transmission of vibrations (e.g., obstructed bronchus, pleural effusion or thickening, pneumothorax, or emphysema). Crepitus is a coarse crackling sensation palpable over the skin surface. It
occurs in subcutaneous emphysema when air escapes from the lung and enters the subcutaneous tissue.
49. A clinical manifestation common in an individual with chronic obstructive pulmonary disease
(COPD) is:
A
periodic breathing patterns.
B
pursed-lip breathing.
C
unequal chest expansion.
D
hyperventilation.

B. An individual with COPD may purse the lips in a whistling position. By exhaling slowly and
against a narrow opening, the pressure in the bronchial tree remains positive, and fewer airways
collapse. Periodic breathing patterns are Cheyne-Stokes or Biot respirations. Cheyne-Stokes
respirations occur in heart failure, renal failure, meningitis, drug overdose, and increased intracranial pressure; this type also normally occurs in infants and aging persons during sleep. Biot
respirations occur with head trauma, brain abscess, heat stroke, spinal meningitis, and encephalitis. Unequal chest expansion occurs when part of the lung is obstructed or collapsed, as with
pneumonia, or when guarding to avoid postoperative incisional pain or pleurisy pain. Hyperventilation is a normal response to fever, fear, or exercise; respiration rate also increases with respiratory insufficiency, pneumonia, alkalosis, pleurisy, and lesions in the pons.
50. The thoracic cage is defined by all of the following except the:
A
sternum.
B
ribs.
C
costochondral junction.
D
diaphragm.

C. The thoracic cage is defined by the sternum, ribs, vertebrae, and diaphragm.

51. Inspiration is primarily facilitated by which of the following muscles?


A
Diaphragm and abdominus rectus
B
Trapezia and sternomastoids
C
Internal intercostals and abdominal
D
Diaphragm and intercostals

D. The major muscle responsible for inspiration is the diaphragm. Intercostal muscles lift the
sternum and elevate the ribs, making them more horizontal; this increases the anteroposterior
diameter.

Page 11 of 81

52. Which of the following voice sounds would be a normal finding?


A
The voice transmission is distinct and sounds close to the ear.
B
The "eeeee" sound is clear and sounds like "eeeee". Correct
C
The whispered sound is transmitted clearly.
D
Whispered 1-2-3 is audible and distinct.
.
B. A normal finding from voice sounds is egophony; eeeee heard through the stethoscope
clearly. A normal finding from voice sounds is whispered pectoriloquy: whispered sound is faint,
muffled, and almost inaudible. A normal finding from voice sounds is bronchophony; normal
voice transmission is soft, muffled, and indistinct.

53. The gradual loss of intraalveolar septa and a decreased number of alveoli in the lungs of the
elderly cause:
A
hyperventilation.
B
spontaneous atelectasis.
C
decreased surface area for gas exchange.
D
decreased dead space.

C. The histologic changes result in less surface area for gas exchange.

54. The function of the trachea and bronchi is to:


A
transport gases between the environment and the lung parenchyma.
B
condense inspired air for better gas exchange.
C
moisturize air for optimum respiration
D
increase air turbulence and velocity for maximum gas transport.

A. The trachea and bronchi transport gases between the environment and the lung parenchyma.
55. Stridor is a high-pitched, inspiratory crowing sound commonly associated with:
A
upper airway obstruction.
B
atelectasis..
C
congestive heart failure.
D
pneumothorax.
A. Stridor is associated with upper airway obstruction from swollen, inflamed tissues or a lodged
foreign body.

56. Percussion of the chest is:


A
a useful technique for identifying small lesions in lung tissue.

Page 12 of 81
B
C
D

helpful only in identifying surface alterations of lung tissue.


is not influenced by the overlying chest muscle and fat tissue.
normal if a dull note is elicited.

B. Percussion detects only the outer 5 to 7 cm of tissue; it will not penetrate to reveal any
change in density deeper than that. An abnormal finding must be 2 to 3 cm wide to yield an abnormal percussion note. Percussion findings (resonant notes) may be modified by a muscular
chest wall of an athlete or subcutaneous tissue of the obese person. Resonance is a lowpitched, clear, hollow sound that predominates with percussion of healthy lung tissue.
57. Which of the following correctly expresses the relationship to the lobes of the lungs and their
anatomic position?
A
Upper lobeslateral chest
B
Upper lobesposterior chest
C
Lower lobesposterior chest
D
Lower lobesanterior chest

C. The posterior chest is almost all lower lobe. The anterior chest contains mostly upper and
middle lobe with very little lower lobe.

58.An increase in the transverse diameter of the chest cage in a pregnant female is due to a(n):
A
compensatory increase in respiratory parenchyma.
B
increase in estrogen.
C
increase in surfactant.
D
increase in tidal volume.
B. The increase in estrogen level during pregnancy relaxes the chest cage ligaments. This allows an increase in the transverse diameter of the chest cage by 2 cm, and the costal angle
widens.
59. Which of the following is a normal finding when assessing the respiratory system of an elderly client?
A.
B.
C.
D.

Bronchovesicular breath sounds throughout the lungs


Increased thoracic expansion and relaxation of elastin tissues
A decreased anteroposterior diameter and increased alveolar surface
Decreased mobility of the thorax and increased chest wall stiffness

D: Decreased mobility of the thorax and increased chest wall stiffness


Rationale: The normal aging process is accompanied by physiologic changes to the respiratory
system such as stiffening of elastin and collagen tissues, increased alveolar diameter, decreased alveolar surface, and increased chest wall stiffness. Bronchovesicular breath sounds
are heard over major bronchi where fewer alveoli are located.

Page 13 of 81

60. An older client is receiving postural drainage treatments but is unable to expel the secretions. The client is confused, and having difficulty following instructions. The best response by
the nurse would be to:
A.
Frequently change the patient's position.
B.
Suction out the secretions.
C.
Have the patient drink water.
D.
Administer humidified oxygen.

B. Suction out the secretions.


Rationale: Postural drainage loosens secretions, which patients usually expectorate when loosened. Since the client is confused, and cannot follow instructions, the nurse must suction the
secretions to enhance effective breathing.
61. All of the following nursing diagnoses are important for a client with chronic pulmonary emphysema (COPD). Which would receive priority when planning nursing interventions?
A.
Impaired gas exchange
B.
Ineffective airway clearance
C.
Activity intolerance
D.
Self-care deficit

A. Impaired gas exchange


Rationale: Physical assessment of older persons with COPD might find ineffective airway clearance, but the classic physiology alteration is the destruction of the alveoli with resulting complications. Therefore, impaired gas exchange always is present.
62. The nurse is teaching a group of older clients about the functions of the respiratory tract as
part of health promotion. Which statement made by patients would indicate a need for further
teaching?
A.
"The lungs keep our blood pressure in normal range, so we can breathe better."
B.
The respiratory system helps maintain heat balance in our bodies."
C.
Parts of our respiratory system help us produce speech and better communication."
D.
Our lungs keep the balance of acids and bases in our bodies."

A. "The lungs keep our blood pressure in normal range, so we can breathe better."
Rationale: The respiratory system performs several secondary functions, including maintenance of acid-base balance, production of speech, and maintenance of body water and heat
balance. The heart and cardiovascular system are responsible for the blood pressure.
63. A seven-month-pregnant female is sitting quietly in the waiting room, and her respiratory
rate is 20 and shallow. What does this finding suggest to the nurse?
A.
She is using accessory muscles to breathe.
B.
Nothing. This is normal.
C.
She has a history of smoking.
D.
She is in pending respiratory failure.

Page 14 of 81
B. Nothing. This is normal.
Rationale: As the uterus enlarges during pregnancy, the pressure in the abdominal cavity also
increases. As a result, diaphragmatic excursion is limited and can result in more rapid and shallow respirations.
64. The nurse is planning to assess the apex of a clients lungs. Which area of the body will the
nurse be assessing?
A.
Left of the sternum, third intercostal space
B.
Above the clavicles
C.
Right of the sternum, sixth intercostal space
D.
Below the scapula
B. Above the clavicles
Rationale: The apex of each lung is slightly superior to the inner third of the clavicle.

65. A client with a strained trapezius muscle complains of having occasional shortness of
breath. What might be the reason for this symptom?
A.
The strained muscle is an accessory muscle of respiration.
B.
The diaphragm muscle is also injured.
C.
There is a blood clot in his lung.
D.
There is an undiagnosed heart problem.
A. The strained muscle is an accessory muscle of respiration.
Rationale: The accessory muscles of the neck (trapezius), abdomen, and chest will assist the
respiratory cycle as necessary.
66. During a physical assessment, the nurse documents eupnea on the clients medical record.
What does this finding suggest?
A.
Irregular respirations
B.
Slow respirations
C.
Rapid respirations
D.
Normal respirations

D. Normal respirations
Rationale: The regular, even-depth, rhythmic pattern of inspiration and expiration describes
eupnea.
67.Prior to listening to a clients lung sounds, the nurse palpates the sternum and feels a horizontal bump on the bone. What does this finding suggest to the nurse?
A.
The manubrium is damaged.
B.
This is the angle of Louis.
C.
The xiphoid process is misshaped.
D.
The costal angle is greater than normal.

B. This is the angle of Louis.


Rationale: The manubrium joints the body of the sternum and forms a horizontal ridge referred
to as the sternal angle or angle of Louis.

Page 15 of 81
68. The nurse is assessing the clients lung bases posteriorly. At which area can the nurse assess this portion of the lung?
A.
Left midclavicular line
B.
Scapular line
C.
Right anterior axillary line
D.
Midsternal line

B. Scapular line
69.The mother of a four-year-old child tells the nurse, I think theres something wrong with him;
his chest is round like a ball. Which of the following would be an appropriate response for the
nurse to make to the mother?
A.
I see what you mean. That seems odd.
B.
The chest of a child appears round and is normal.
C.
I wouldnt worry about that.
D.
Did you tell the doctor about this?
B. The chest of a child appears round and is normal. It is this way till about six years old considered normal.
70. After examining a 75-year-old male client, the nurse writes down barrel chest. What does
this finding suggest?
A.
The client has osteoporosis.
B.
The client has a history of smoking.
C.
This is a change associated with aging.
D.
The client has long-standing respiratory disease.

C. This is a change associated with aging.


Rationale: As individuals age, body functions change. The diameters of the thoracic cage
change. The appearance of a barrel chest (kyphosis) can contribute to the decrease in thoracic
excursion.
71. A 57-year-old client tells the nurse, I need two to three pillows to sleep. How should this
information be documented?
A.
Dyspnea at rest
B.
Two to three pillow orthopnea
C.
Resting apnea
D.
Dyspnea on excursion

B. Two to three pillow orthopnea


Rationale: Clients who must prop themselves up or sit up while sleeping might have orthopnea,
that is, dyspnea when lying down.
72. The client tells the nurse he sometimes coughs up thick yellow mucous. What does this
information suggest to the nurse?
A.
He might have a fungal infection.
B.
He might have episodic lung infections.

Page 16 of 81
C.
D.

He might have an allergy.


He might have tuberculosis.

B. He might have episodic lung infections.


73. A 48-year-old client doesnt smoke cigarettes yet is demonstrating signs of lung irritation.
Which of the following questions could help with the assessment of this client?
A.
Have you tried to stop smoking?
B.
Have you received a flu or pneumonia vaccination?
C.
Do you smoke or inhale marijuana or other herbal products?
D.
Have you had allergy testing?

C. Do you smoke or inhale marijuana or other herbal products?


Rationale: Inhalation of marijuana, herbal substances, and/or chemicals may result in respiration problems associated with incidental or continuous irritation of the linings of the respiratory
organs.
74. After inspecting a clients thorax, the nurse writes AP:T 1:2, bilateral symmetrical movements, sternum midline, respiratory rate 16 and regular. What do these findings suggest?
A.
The client has a respiratory illness.
B.
The client has pneumonia.
C.
Nothing. These findings are normal.
D.
The client has allergies.

C. Nothing. These findings are normal.


Rationale: Normal assessment findings include an AP:T diameter of 1:2, bilateral symmetrical
chest movements, a midline sternum, and a respiratory rate between 12 to 20.
75.While palpating the posterior thorax of a client, the nurse notes increased fremitus. What
does this finding suggest to the nurse?
A.
The client needs to speak up.
B.
The client has a thick chest wall.
C.
The client could either have fluid in the lungs or have an infection.
D.
Nothing. This is a normal finding.

C. The client could either have fluid in the lungs or have an infection.
76.The nurse sees that the client will breathe deeply and then stop breathing for a short while.
Which of the following does this observation suggest?
A.
This client is hyperventilating.
B.
This is seen in aging people, people with heart failure, and people who have suffered brain damage.
C.
This client is in a diabetic coma.
D.
This client has pneumonia.

Page 17 of 81
B. This is seen in aging people, people with heart failure, and people who have suffered brain
damage.
Rationale: Cheyne-Stokes respirations are periods of deep breathing alternating with periods of
apnea. Precipitating factors include aging, heart failure, uremia, brain damage, and druginduced respiratory depression.
77. In planning a patient education session, the nurse sees one area of focus for Healthy People 2010 is chronic obstructive pulmonary disease (COPD). Which of the following information
should the nurse include in the education session to address this focus area?
A.
Develop action plans
B.
Smoking cessation
C.
Screening for environmental triggers
D.
Identify those at risk

B. Smoking cessation
Rationale: One of the actions for the focus area COPD is smoking cessation. Others include
screening for COPD and counseling or treatment.
78. A client asks the nurse, Why do I need to examine my armpits when I do my monthly breast
exam? Which of the following would be an appropriate response for the nurse to make to this
client?
A.
Breast tissue extends into the axilla.
B.
Im not sure why that is important, but it sounds like it is.
C.
Who told you that you have to do that?
D.
This is the hardest area to feel for changes.

A. Breast tissue extends into the axilla


79. During the breast exam, the nurse palpates a series of lymph nodes. Why is this a part of
the breast exam?
A.
Its not. Its done because the chest area is exposed.
B.
To assess shoulder range of motion.
C.
To assess the deep lymph nodes which drain the mammary lobules.
D.
To review the integrity of the skin.

C. To assess the deep lymph nodes which drain the mammary lobules.
80. A 14-year-old female client is upset because her breast development is not equal. What can
the nurse say to this client?
A.
Breast tissue growth is uneven but will even out as you get older.
B.
They look equal to me.
C.
Maybe you should talk with your mother about breast surgery?
D.
Dont worry about that!

A. Breast tissue growth is uneven but will even out as you get older.
81. An elderly female client wants to know when she can stop doing breast exams. What can
the nurse say to this client?

Page 18 of 81
A.
B.
C.
D.

You can stop five years after menopause.


Breast cancer can still develop when you get older.
Probably in a month or two.
Its not really necessary at your age.

B. Breast cancer can still develop when you get older.


82. A female client tells the nurse, I know I should examine my breasts but I just dont. What
should the nurse do with this information?
A.
Instruct the client on getting an annual mammogram instead.
B.
Talk with the client about possible fears associated with the breast exam.
C.
Instruct the client on how to perform the breast exam again.
D.
Nothing.
B. Talk with the client about possible fears associated with the breast exam.
83. The nurse is planning a focused breast/axilla interview and wants to include a general health
question. Which of the following questions would fit this criteria?
A.
Are you still menstruating?
B.
Have you had any breast trauma?
C.
Has your mother or sister had breast cancer?
D.
Have you ever had a mammogram?

A. Are you still menstruating


Rationale: General health questions for the breast/axilla focused interview include a description
of the breasts, changes in the breasts with menstruation, and date of the last menstrual period.
84.A 58-year-old client says to the nurse, My saggy breasts embarrass me! What can the
nurse say to this client in response?
A.
Breasts sag because of declining estrogen levels.
B.
Maybe you can have breast augmentation surgery?
C.
Dont be silly.
D.
Wearing a good bra will help.

A. Breasts sag because of declining estrogen levels.


85. A pregnant client is upset and thinks she has breast disease because she has a thick white
discharge coming from her left breast. What can the nurse say or do for this client?
A.
Help the client understand that she might not be able to breastfeed her infant.
B.
A thick yellow discharge from the breasts during pregnancy is normal.
C.
Call the physician. This information is not normal.
D.
Nothing. This client needs a mammogram as soon as possible.

B. A thick yellow discharge from the breasts during pregnancy is normal.


86. A client comes into the clinic for a routine breast and axilla exam. Which assessment technique does the nurse use first during this examination?
A.
Palpation

Page 19 of 81
B.
C.
D.

Inspection
Percussion
Auscultation

B. Inspection
87. During the breast exam, the nurse asks the client to raise her arms over her head. Why did
the nurse change the clients position?
A.
The client has small breasts.
B.
The nurse couldnt palpate the axillae correctly.
C.
The client has large breasts.
D.
Skin dimpling is accented in this position.
D. Skin dimpling is accented in this position.
88. The nurse is examining a clients breasts and follows a specific pattern. Which of the following patterns can be used for a breast exam?
A.
Letter S
B.
Back and forth technique.
C.
Letter H
D.
ABC

B. Back and forth technique


89. The client tells the nurse, At times I have drainage from my right breast. What should the
nurse do with this information?
A.
Nothing. It doesnt mean a thing.
B.
Write it in the medical record and say nothing to the client.
C.
Phone for a mammogram for the client immediately.
D.
Explain that this could be benign or it could mean something else. It needs to be
further investigated.

D. Explain that this could be benign or it could mean something else. It needs to be further investigated.
90. A client with fibrocystic breast disease says she has increased breast pain and tenderness
with menses. Which of the following can help this client?
A.
Review how reducing caffeine and salt and wearing a support bra might help.
B.
Discuss how this can be a precursor to breast cancer.
C.
Nothing can be done to help this client.
D.
Explain how a breast biopsy is indicated.

A. Review how reducing caffeine and salt and wearing a support bra might help.
Rationale: In benign breast disease, some studies suggest limiting caffeine, decreasing salt
intake, and taking a mild analgesic might help with the breast pain. Wearing a support bra also
can decrease discomfort.

Page 20 of 81
91.The clinic is sponsoring a client education session for breast cancer awareness month.
Which of the following considerations should be included to support cultural differences about
breast health?
A.
Inform all about the low-cost breast cancer screening program.
B.
Encourage all females to complete monthly breast exams.
C.
Refer all clients to the American Cancer Society if they have questions.
D.
Encourage all females to increase their intake of vitamins A and E.
B. Encourage all females to complete monthly breast exams.
Rationale: One cultural consideration for the instructions associated with breath/axilla health
include recognizing that some female clients might be uneasy touching their own breasts but
should be encouraged to do so and seek care regularly for breast examination.
92. A postmenopausal client has difficulty remembering to complete a monthly breast exam.
What can be done to help this client?
A.
Schedule the client to come into the clinic every month for the exam.
B.
Nothing. This client doesnt need to do self examinations.
C.
Schedule the client to receive a monthly phone call from the clinic as a reminder.
D.
Suggest the client plan to conduct the exam the first day of every month.

D. Suggest the client plan to conduct the exam the first day of every month.

93. The first heart sound is produced by the:


A
closure of the semilunar valves.
B
closure of the AV valves.
C
opening of the semilunar valves.
D
opening of the AV valves.

B. The first heart sound (S1) occurs with closure of the AV valves. The second heart sound (S2)
occurs with closure of the semilunar valves. Normally opening of the semilunar valves is silent,
but in aortic or pulmonic stenosis, an ejection click may be heard. An ejection click occurs early
in systole at the start of ejection because it results from opening of the semilunar valves. A third
heart sound (S3) can be heard when the ventricles are resistant to filling during the early rapid
filling phase. S3 is heard when the AV valves open and atrial blood first pours into the ventricles.

94. Which of the following guidelines may be used to identify which heart sound is S1?
A
S1 is louder than S2 at the base of the heart.
B
S1 coincides with the A wave of the jugular venous pulse wave.
C
S1 coincides with the carotid artery pulse.
D
S1 coincides with the Q wave of the QRS electrocardiogram complex.

C S1 coincides with the carotid artery pulse. S1 is loudest at the apex of the heart. S1 coincides
with the C wave of the jugular venous pulse wave S1 coincides with the R wave (the upstroke of
the QRS complex).

Page 21 of 81
95. Which of the following cardiac alterations occurs during pregnancy?
A
An increase in cardiac output and blood pressure
B
An increase in cardiac volume and a decrease in blood pressure
C
An increased heart rate and increased blood pressure
D
An increased stroke volume with decreased cardiac output

B. During pregnancy the blood volume increases by 30% to 40%; this creates an increase in
stroke volume and cardiac output and an increased pulse rate of 10 to 15 beats per minute. The
arterial blood pressure decreases in pregnancy as a result of peripheral vasodilation.
96. Which of the following is an appropriate position to have the patient assume when auscultating for extra heart sounds or murmurs?
A
Roll toward the left side
B
Roll toward the right side
C
Trendelenburg position
D
Recumbent position

A After auscultation in the supine position, the nurse should have the patient roll onto the left
side; the examiner should listen at the apex with the bell for the presence of any diastolic filling
sounds (i.e., S3 or S4) or murmurs that may be heard only in this position. The examiner should
have the patient sit up and lean forward; the examiner should auscultate at the base with the
diaphragm for a soft, high-pitched, early diastolic murmur of aortic or pulmonic regurgitation.

97. The leaflets of the tricuspid and mitral valves are anchored by __________________ to the
_________________, which are embedded in the ventricular floor.
A
endocardial ligaments; mediastinal muscles
B
atrioventricular tendons; pericardial bundles
C
chordae tendineae; papillary muscles
D
pericardial cords; ventricular sheaths

C The valves are anchored by collagenous fibers (chordae tendineae) to the papillary muscles,
which are embedded in the ventricle floor.
98. The ability of the heart to contract independently of any signals or stimulation is due to:
A
depolarization.
B
automaticity.
C
conduction.
D
repolarization.

B The heart can contract by itself, independent of any signals or stimulation from the body; this
property is termed automaticity. Depolarization is the reversal of the resting potential in excitable
cardiac muscle cell membranes when stimulated. Conduction is the process by which an electrical impulse is transmitted through the heart. Repolarization is the process by which the membrane potential of a cardiac muscle cell is restored to the cell's resting potential.
99. When auscultating the heart of a newborn within 24 hours after birth, the examiner hears a
continuous sound that mimics the sound of a machine. This finding most likely indicates:

Page 22 of 81
A
B
C
D

the presence of congenital heart disease.


a normal sound because of the thinner chest wall of the newborn.
an expected sound caused by nonclosure of the ductus arteriosus.
pathology only when accompanied by an increased heart rate.

C The murmur of a patent ductus arteriosus is a continuous machinery murmur, which disappears by 2 to 3 days.
100. A bruit heard while auscultating the carotid artery of a 65-year-old patient is caused by:
A
decreased velocity of blood flow through the carotid artery.
B
turbulent blood flow through the carotid artery.
C
rapid blood flow through the carotid artery.
D
increased viscosity of blood.

B. A carotid bruit is a blowing, swishing sound indicating blood flow turbulence. A bruit indicates
atherosclerotic narrowing of the vessel.

Incorrect Awarded 0.0 points out of 1.0 possible points.

101. The jugular venous pressure is an indirect reflection of the:


A
heart's efficiency as a pump.
B
cardiac cycle.
C
conduction effectiveness.
D
synchronization of mechanical activity.
A. Jugular venous pressure is a reflection of the hearts ability to pump blood. If the pressure is
elevated, heart failure is suspected.
102. The semilunar valves separate the:
A
atria from the ventricles.
B
right atria from the left atria.
C
ventricles from the arteries.
D
atria from the veins.

C. The semilunar valves separate the ventricles from the arteries. The atrioventricular valves
separate the atria and ventricles. The septum separates the right atria from the left atria. The
vena cava are not separated by a valve from the right atrium; the pulmonary veins are not separated by a valve from the left atrium.
103. A client has been diagnosed with pericarditis. Which layers of the heart are affected with
this illness?
A.
Between the myocardium and the mediastinum
B.
Between the pericardium and the myocardium
C.
Between the myocardium and the endocardium

Page 23 of 81
D.

Between the pericardium and the mediastinum

B. Between the pericardium and the myocardium


Rationale: The pericardial space is between the pericardium and the myocardium. This is the
location where infection can occur and lead to pericarditis.
104. The nurse is listening to a clients heartbeat and is focusing on the second heart sound.
Which heart valves produce this sound?
A.
Mitral and pulmonic
B.
Aortic and tricuspid
C.
Aortic and pulmonic
D.
Mitral and tricuspid

C. Aortic and pulmonic


Rationale: The second heart sound is produced with the closure of the aortic and pulmonic
valves.
105. The nurse hears a heart sound right before S1 on a 72-year-old male client. What can this
finding suggest to the nurse?
A.
This is an atrial kick and helps the heart beat better.
B.
This is an atrial gallop and can mean something is wrong.
C.
This is a ventricular gallop and is heard in healthy people.
D.
Nothing. This is normal.

B. This is an atrial gallop and can mean something is wrong.


Rational: An S4 heard before S1 is termed an atrial gallop and may be associated with pathologic conditions such as myocardial infarction or heart failure.
106. The nurse hears a heart murmur on a client and wants to grade this soft but very audible
sound. How should this heart murmur be described?
A.
Grade 4
B.
Grade 2
C.
Grade 1
D.
Grade 3
B. Grade 2
Rationale: A grade 2 heart murmur is described as being very soft but distinctly audible.
107. A client has been diagnosed with a myocardial infarction that has damaged a part of the
right atrium. Which of the following could happen as a result of this damage?
A.
Onset of aortic regurgitation
B.
Nothing
C.
A change in the rate and rhythm of the clients heartbeat
D.
Increase in collateral circulation
C. A change in the rate and rhythm of the clients heartbeat
Rationale: The sinoatrial node initiates the electrical impulse and is termed the pacemaker of
the heart. This node is located at the junction of the superior vena cava and right atrium.

Page 24 of 81

108. A nurse, explaining the cardiac circulation to cardiac rehabilitation clients, wants to include
the oxygenation of the heart muscle. Which of the following structures carries deoxygenated
blood to the lungs?
A.
Great cardiac vein
B.
Pulmonary vein
C.
Right main coronary vein
D.
Pulmonary artery

D. Pulmonary artery
Rationale: The pulmonary artery is the only artery to carry deoxygenated blood.
109. The nurse is calculating the cardiac output for a client with a heart rate of 88 beats per minute. What other measurements does the nurse need to complete this calculation?
A.
Cardiac index
B.
QT interval
C.
Stroke volume
D.
T wave
C. Stroke volume
Rationale: To calculate the cardiac output, the stroke volume is multiplied by the heart rate.
110. The nurse finds a 76-year-old clients point of maximum impulse farther away from the fifth
intercostal space, midclavicular line. What does this finding suggest to the nurse?
A.
The client had pneumonia.
B.
The client has benign prostatic hypertrophy.
C.
The client had a heart attack.
D.
The client has hypertension.
D. The client has hypertension
Rationale: Significant enlargement of the left ventricle can be attributed to the influence of hypertension.
111. During the cardiac focused interview, the client tells the nurse he has smoked for 30 years.
Which of the following would be appropriate for the nurse to respond to this client?
A.
Smoking has been linked to high blood pressure and other heart problems.
B.
Do you also take recreational drugs?
C.
High dose vitamin therapy reduces the risks of smoking.
D.
Exercise reduces the harmful effects of smoking.

A. Smoking has been linked to high blood pressure and other heart problems.
Rationale: Smoking has been linked to hypertension, atherosclerotic plaques, and coronary artery disease.

Page 25 of 81

112. During the examination of an eight-month-pregnant client, the nurse measures the blood
pressure at 160/98 and notes bilateral edema of the ankles. What do these findings suggest to
the nurse?
A.
This client is going into labor early.
B.
The client could be preeclamptic.
C.
The client is in normal health.
D.
The client is hyperventilating.

B. The client could be preeclamptic.


Rationale: Hypertension and swelling are symptoms of preeclampsia and can place the mother
and infant at risk if not treated.
113. During the cardiac assessment, the nurse finds a client has jugular vein distention. What
does this mean to the nurse?
A.
The client has an infection.
B.
The client is dehydrated.
C.
The client could have fluid overload.
D.
The client is fine.

C. The client could have fluid overload.


Rationale: Distention of the neck veins indicates elevated central venous pressure commonly
seen with congestive heart failure, fluid overload, or pressure on the superior vena cava.
114.The nurse feels pulsations on a clients right sternal border, second space. What does this
finding suggest to the nurse?
A.
Tricuspid valve regurgitation
B.
Aortic stenosis
C.
A prolapsed mitral valve
D.
Nothing. This is a normal finding.
B. Aortic stenosis
Rationale: Pulsations or heaves in the right sternal border, second intercostal space indicate the
presence of ascending aortic enlargement or aneurysm, aortic stenosis, or systemic hypertension.
115. During the cardiac assessment, the nurse learns a client had rheumatic heart fever as a
child. For which of the following cardiac conditions should this client be assessed?
A.
Mitral regurgitation
B.
Pulmonic stenosis
C.
Mitral stenosis
D.
Aortic stenosis

C. Mitral stenosis
Rationale: Mitral stenosis, the narrowing of the mitral valve, can be caused by rheumatic fever
or another cardiac infection.

Page 26 of 81
116. The nurse is instructing a client about his high cholesterol level and wants to include behavioral considerations. Which of the following should be included in this instruction?
A.
The need for an annual cholesterol panel
B.
The need for annual flu inoculation
C.
The need to stop smoking
D.
The need to reduce stress

C. The need to stop smoking


Rationale: Behavioral considerations to include in this instruction include smoking cessation and
the need for regular exercise.

117. One of the leg's deep veins is the:


A
great saphenous.
B
small saphenous.
C
tibial.
D
popliteal.
D. The femoral and popliteal veins are the deep veins in the leg. The anterior tibial veins extend
downward from the popliteal veins. The superficial veins are the great and small saphenous
veins. The superficial veins are the great and small saphenous veins.

118. Arteriosclerosis refers to:


A
a variation from the heart's normal rhythm.
B
a sac formed by dilation in the arterial wall.
C
thickening and loss of elasticity of the arterial walls.
D
deposition of fatty plaques along the intima of the arteries.
C. Arteriosclerosis is the thickening and loss of elasticity of the arterial walls. Atherosclerosis is
the deposition of fatty plaques on the intima of the arteries. An aneurysm is a sac formed by dilation in the artery wall. A dysrhythmia is a variation from the hearts normal rhythm.
.
119. Palpable inguinal lymph nodes are:
A
normal if small (less than 1 cm), movable, and nontender.
B
abnormal in adults but common in children and infants..
C
normal if fixed and tender.
D
abnormal and indicate the presence of malignant disease.

A. Inguinal lymph nodes may be palpable. This is a normal finding if the nodes are small (1 cm
or less), movable, and nontender. Lymph nodes may be relatively large in children, and the
superficial ones often are palpable even when the child is healthy. Enlarged, tender, or fixed
inguinal lymph nodes are an abnormal finding.

120. In pulsus paradoxus:

Page 27 of 81
A
the rhythm is irregular; every other beat is premature.
B
there is a deficiency of arterial blood to a body part.
C
the rhythm is regular, but the force of the pulse varies with alternating beats.
D
beats have weaker amplitude with respiratory inspiration and stronger amplitude
with expiration.

D. In pulsus paradoxus beats have weaker amplitude with inspiration and stronger amplitude
with expiration. In pulsus alternans the rhythm is regular, but the force varies with alternating
beats of large and small amplitude. A weak, thready pulse may result in a deficiency of arterial
blood to a body part. In pulsus bigeminus the rhythm is irregular and coupled, every other beat
comes early or premature.
121. Lymphedema is:
A
the indentation left after the examiner depresses the skin over swollen edematous tissue.
B
a thickening and loss of elasticity of the arterial walls.
C
an inflammation of the vein associated with thrombus formation.
D
the swelling of an extremity caused by an obstructed lymph channel.

D. Lymphedema is swelling of the limb caused by surgical removal of lymph nodes or damage
to lymph nodes and vessels. n deep vein thrombosis a deep vein is occluded by a thrombus,
causing inflammation, blocked venous return, cyanosis, and edema. Arteriosclerosis is the
thickening and loss of elasticity of the arterial walls. Pretibial edema (pitting) occurs if an indentation is left after the examiner depresses skin over the tibia or the medial malleolus for 5 seconds.
122. A water-hammer "Corrigan" pulse is associated with:
A
hyperkinetic states.
B
decreased cardiac output.
C
aortic valve regurgitation.
D
conduction disturbance.

C. A water-hammer (Corrigan) pulse occurs in aortic valve regurgitation. A weak, thready pulse
occurs with decreased cardiac output. A full, bounding pulse is associated with hyperkinetic
states (exercise, anxiety, fever). Pulsus bigeminus occurs with conduction disturbances.

123. The cervical nodes drain the:


A
upper arm and breast.
B
hand and lower arm.
C
external genitalia.
D
head and neck.

Page 28 of 81

D. The cervical nodes drain the head and neck. Inguinal nodes drain most of the lymph from the
lower extremity, the external genitalia, and the anterior abdominal wall. The epitrochlear node
drains the hand and lower arm. Axillary nodes drain the breast and upper arm.
124. In young children, the thymus gland:
A
produces T lymphocytes.
B
is small and begins to atrophy.
C
is not important in immune function.
D
produces B lymphocytes.

A. In young children, the thymus gland is important in developing the T lymphocytes of the immune system. The thymus is large in the fetus and young children and atrophies after puberty.
The thymus has no function in adults. The thymus gland does not produce B lymphocytes.
125. Claudication is caused by:
A
venous insufficiency.
B
arterial insufficiency.
C
varicose veins.
D
stasis ulcerations.

A. Claudication is caused by arterial insufficiency. Varicose veins are venous in origin. Stasis
ulcerations are venous in origin.
126. The patient has severe bilateral lower extremity edema. The most likely cause is:
A
an infection of the right great toe.
B
Raynaud phenomenon.
C
heart failure.
D
an aortic aneurysm.
B. Bilateral lower extremity edema is a result of a generalized disorder such as heart failure.
Aneurysms do not cause bilateral lower extremity edema. An infection of only one extremity
would lead to unilateral edema. Raynaud phenomenon does not result in bilateral lower extremity edema.

127. A client is recovering from a cardiac catheterization where the right femoral artery was accessed. Which of the following pulses can the nurse use to assess the patency of this artery?
A.
Radial
B
Anterior tibial
C
Brachial
D
Ulnar

B. Anterior tibial
Rationale: The femoral artery continues down the thigh and becomes the popliteal artery. The
popliteal artery divides into the anterior and posterior tibial arteries.

Page 29 of 81
128. The nurse assesses ankle edema in a client. What can the nurse say to the client about
this edema?
A.
Its caused by an infection.
B.
Its caused by blood pooling in the legs.
C.
Its caused by tight stockings.
D.
Its caused by a blood clot.

C. It is caused by blood pooling in the klegs

129. The neonatal nurse obtains a newborns blood pressure as being 76/40 mm Hg. Which of
the following should the nurse do with this information?
A.
Call the physician.
B.
Write it down. This is a normal neonatal blood pressure.
C.
Call for help as the baby is going to cardiac arrest.
D.
Ask another nurse to check the blood pressure because it is low.

B.

130. An eight-month-pregnant client is worried about all of the swollen veins she has developed.
What can the nurse tell the client about these veins?
A.
Its too bad that your legs look like that.
B.
Its because of the uterus causing blood to back up.
C.
Maybe you should see a vascular surgeon.
D.
Nothing. Phone the physician and report the finding.

B.
131. During the assessment of a clients blood pressure, the nurse notices the client starting to
cry. What should the nurse do?
A.
Nothing.
B.
Offer a paper tissue.
C.
Begin talking about the weather as a distraction to the client.
D.
Stop taking the blood pressure.

132. A client tells the nurse, My legs are always cold. What significance does this information
have to the nurse?
A.
The client might have arterial insufficiency.
B.
The client smokes.
C.
The client has edema.
D.
The client has stasis ulcers.

Page 30 of 81

A.
133. A 47-year-old female tells the nurse, Im sorry for my hairy legs. I shaved a few days ago
and didnt have time this morning to do it again. What does this information mean to the nurse?
A.
The client needs instruction on safe hair removal.
B.
The client has good peripheral extremity blood flow.
C.
The client has personal hygiene issues.
D.
The client needs to plan more time for self-care in the morning.

B.
134. A client tells the nurse, My legs really hurt when I walk. What can the nurse say to this
client in response?
A. Maybe you have a lymph node infection in your groin.
B. Maybe you shouldnt walk as much and see if it improves.
C. That means you have to walk more.
D. Does the pain go away when you stop walking?

D.
135. A 33-year-old female client comes into the clinic with an edematous left calf that is painful
to the touch, feels warm, and is red. Which of the following should be included in the focused
interview of this client?
A.
How long have you had varicose veins?
B.
Are you experiencing any emotional upset?
C.
Have you had any facial swelling?
D. What medications are you taking?

D.
136. The nurse wants to further assess a clients radial pulse. What can be done to do this assessment?
A.
Conduct the Trendelenburgs test.
B.
Conduct the Allens test.
C.
Conduct the manual compression test.
D.
Conduct the Babinski test.
B. Conduct the Allens test.
Rationale: The Allens test determines the patency of the radial and ulnar arteries.
137. A clients blood pressure is 158/90 mm Hg. What does this reading suggest to the nurse?
A.
This client has stage 1 hypertension.
B.
This client has prehypertension.
C.
This client has stage 2 hypertension.
D.
This blood pressure is normal.

Page 31 of 81
A. This client has stage 1 hypertension.
Rationale: In stage 1 hypertension, the systolic reading is between 140 to 159 mm Hg and diastolic reading between 90 to 99 mm Hg.
138. A clients fingertips are large and very round. How can this data be documented?
A.
Edematous finger tips
B.
Fingertips large and round
C.
Oversized hands
D.
Possible clubbing and turkey drumstick appearance of fingertips
D. Possible clubbing and turkey drumstick appearance of fingertips
Rationale: Flattening of the angle of the nail and enlargement of the tips of the fingers is termed
clubbing. A rounding of the tip of the finger is described as turkey drumsticks.
139. A client tells the nurse she has calf pain. Which of the following should the nurse do?
A. Call the physician and prepare for a heparin infusion.
B. Ask the client to walk around the examination room.
C. Walk with the client and ask her to rate the pain.
D. Pull up on the clients toes to see if theres more pain.

D. Pull up on the clients toes to see if theres more pain.


Rationale: A positive Homans sign could indicate a blood clot in the leg. This sign is unreliable
and follow-up studies may be required to identify the presence of a clot.

140. A 68-year-old client has lower extremity edema and thick skin discolored to a dark brown.
The client complains of his legs feeling full. Which of the following does this information suggest to the nurse?
A.
The client might have venous insufficiency.
B.
The client has a deep vein thrombosis.
C.
The client has varicose veins.
D.
The client might have an arterial blood clot.
A. The client might have venous insufficiency.
Rationale: Venous insufficiency is inadequate circulation in the venous system.
Edema is usually present and is accompanied by a feeling of leg fullness. The skin
around the ankles may be thicker and have a brown discoloration.
141. Using the Healthy People 2010 recommendations for blood pressure and stroke, the nurse
wants to include actions for both of these conditions in the next staff inservice. Which of the following actions would be appropriate for both of these conditions?
A.
Exercise
B.
Warning signs
C.
Smoking cessation
D.
Annual blood pressure screening

C.Smoking cessation

Page 32 of 81
Rationale: Smoking cessation is an appropriate action for the focus areas of high blood pressure and stroke.
142. A client tells the nurse, I have a stomachache almost every day. Which of the following
could the nurse respond to this client?
A.
Thats not a good sign.
B.
I would see a surgeon if I were you.
C.
Maybe we can talk about your diet.
D.
Thats too bad.

C.

143. The nurse learns a client takes more than three times the recommended amount of an over
the counter pain reliever on a daily basis and is concerned about his liver. What is the reason
for this concern?
A.
The patient is altering the bodys ability to produce insulin.
B.
Maybe the client would benefit from a prescribed pain reliever.
C.
The patient is altering the bodys ability to digest fats.
D.
The liver might become damaged.

D. The liver might become damaged.


Rationale: The liver has the ability to store some vitamins, produce substances for coagulation
of blood, produce antibodies, and detoxify harmful substances.

144. The nurse is preparing to examine a clients abdomen. Which of the following landmarks
could be considered a thoracic structure?
A.
Xiphoid process
B.
Iliac crests
C.
Pubic bone
D.
Umbilicus
A. Xiphoid process
Rationale: The xiphoid process is the distal end of the sternum and typically thought of as being
a thoracic structure. The other structures are within the boundaries of the abdomen.

145. The nurse wants to assess bowel sounds on a client with abdominal pain. Where would the
nurse hear those sounds the best coming from the stomach?
A.
LUQ
B.
LLQ
C.
RLQ
D.
RUQ

A. LUQ

Page 33 of 81
Rationale: This quadrant includes the structure of the stomach and would most likely provide the
best sounds for this organ.
146. The nurse is planning to palpate a clients bladder. Which area of the abdomen should this
palpation be done?
A.
Right hypochondriac region
B.
RLQ
C.
Left lumbar region
D.
Hypogastric region

D. Hypogastric region
Rationale: The hypogastric region contacts the bladder.
147. The mother of an 18-month-old child tells the nurse, I can see his belly rumbling. Is this
normal? Which of the following can the nurse respond to this client?
A.
No. This is not normal.
B.
There is a good pediatric gastroenterologist that I know who can help you.
C.
This means his gallbladder is digesting fats.
D.
The muscles of the abdomen are thin in babies. So you will see this.

D. The muscles of the abdomen are thin in babies. So you will see this.
Rationale: Peristaltic waves are usually more visible in infants and children than in adults because the muscle wall of the abdomen is thinner.
148. The 79-year-old female tells the nurse, I dont drink as much water as I should because it
makes me have to go to the bathroom. What is this client prone to developing?
A.
Diarrhea
B.
Constipation
C.
Hemorrhoids
D.
Acid indigestion

B.
Constipation
Rationale: Constipation is a common problem with older adults. The older adult may self-limit
the daily fluid intake to decrease frequency of urination with the increased potential for constipation.
149. An overweight client tells the nurse, I was raised to eat everything on my plate. What can
the nurse say to help with this clients weight problem?
A.I was raised the same way.
B.I know; its not worth the effort to fight a losing battle.
C.Nothing. This is none of the nurses business.
D.There are some small changes that you can make to help with your weight problem.
D. There are some small changes that you can make to help with your weight problem.
Rationale: Clients who perceive themselves as naturally fat may overeat and avoid exercise,
feeling that there is nothing they can do to alter their weight.

Page 34 of 81
150. After inspecting a clients abdomen, which assessment technique should the nurse use
next?
A.
Nothing. The assessment is complete.
B.
Palpation
C.
Percussion
D.
Auscultation

D. Auscultation
151. A client comes to the hospital with nausea, vomiting, and ongoing sciatic pain. Which of the
following should be included in the focus interview with this client?
A.
Blood pressure levels
B.
Use of pain medication
C.
Review of other chronic diseases
D.
Bowel habits
B. Use of pain medication
Rationale: Medications may contribute to or cause vomiting. Since the client has sciatic pain, it
would wise for the nurse to assess what types of medications have been used to control this
pain in efforts to find the cause for the nausea and vomiting.
152.A client tells the nurse, I get stomach burning when I drink wine. Of what is this information an indication?
A.
Gallstones
B.
Intestinal ulcerations
C.
Gastrointestinal irritation
D.
Stomach bleeding

C. Gastrointestinal irritation
Rationale: Caffeine and alcohol irritate the gastrointestinal system and can contribute to ulcers
and irritable bowel syndrome.
153. The client tells the nurse, Ive had diarrhea ever since my mother was admitted to the
hospital with a heart attack. What can the nurse say to the client about this information?
A.
Hows your mother doing now?
B.
What hospital is your mother in?
C.
Stress can cause the bowels to act up.
D.
Are you having any other problems?

C. Stress can cause the bowels to act up.


Rationale: Stress is linked to gastrointestinal disease.
154. After assessing a client, the nurse writes striae present bilateral costal margins. What
should the nurse do with this information?
A.
Ask the client if theyve experienced any recent emotional events.
B.
Nothing. This is a normal finding.
C.
Notify the physician.
D.
Suggest the client see a general surgeon.

Page 35 of 81

B.

155. The nurse auscultates borborygmi on a client. What does this finding indicate to the nurse?
A.
The client is anorexic.
B.
The client is obese.
C.
The client had a full breakfast.
D.
The client is hungry.

D. The client is hungry.


Rationale: Borborygmi (stomach growling) refers to more frequent sounds heard in clients who
have not eaten for a few hours.
156. A 70-year-old male client comes into the clinic with weight loss and difficulty swallowing.
Which of the following should the nurse document for this client?
A.
Bulimia
B.
Odynophagia
C.
Dysphagia
D.
Aphasia

D. Dysphagia
Rationale: Dysphagia is defines as difficulty swallowing.

157. Ascites is defined as:


A
a bowel obstruction.
B
a proximal loop of the large intestine.
C
an abnormal enlargement of the spleen.
D
an abnormal accumulation of serous fluid within the peritoneal cavity.

D. Ascites is free fluid in the peritoneal cavity. Splenomegaly is the term to describe an enlarged
spleen. The proximal loop of the large intestine is the ascending colon. A bowel obstruction may
result in abdominal distention.
158. Pyloric stenosis is a(n):
A
abnormal enlargement of the pyloric sphincter.
B
inflammation of the pyloric sphincter.
C
congenital narrowing of the pyloric sphincter.
D
abnormal opening in the pyloric sphincter.

C. Pyloric stenosis is a congenital defect causing a narrowing of the pyloric sphincter.

Page 36 of 81
159. Moles on the abdomen:
A
are common.
B
are uncommon.
C
require a biopsy.
D
are no cause for concern.

A. Pigmented nevi (moles) are common on the abdomen. Nevi are circumscribed brown macular or popular areas. Nevi should be observed for unusual color or change in shape; a biopsy or
removal is indicated if nevi changes, which indicates a possible malignancy.

160. The organ in the right upper quadrant of the abdomen is the:
A
spleen.
B
liver.
C
cecum.
D
sigmoid colon.

B. The liver is in the right upper quadrant of the abdomen. The sigmoid colon is in the left lower
quadrant. The cecum is in the right lower quadrant. The spleen is in the left upper quadrant.

161.The four layers of large, flat abdominal muscles form the:


A
linea alba.
B
rectus abdominus.
C
ventral abdominal wall.
D
viscera.
C. The four layers of large, flat muscles form the ventral abdominal wall. The viscera are all the
internal organs inside the abdominal cavity. One set of abdominal muscles, the rectus abdominis, forms a strip extending the length of the midline. These muscles are joined at the midline by
a tendinous seam, the linea alba.

162. Pyrosis is:


A
an inflammation of the peritoneum.
B
a burning sensation in the upper abdomen.
C
a congenital narrowing of the pyloric sphincter.
D
an abnormally sunken abdominal wall.
B. Pyrosis (heartburn) is a burning sensation in the esophagus and stomach from reflux of gastric acid. Peritonitis is an inflammation of the peritoneum. Pyloric stenosis is a congenital narrowing of the pyloric sphincter. A scaphoid abdomen abnormally caves in or is sunken.
163. The abdomen normally moves when breathing until the age of ____ years.
A
4
B
7
C
14
D
75

Page 37 of 81

B. Abdominal breathing in children continues until the age of 7 years.


164. Older adults have:
A
decreased salivation leading to dry mouth.
B
increased gastric acid secretion.
C
increased liver size. Aging results in decreased liver size.
D
decreased incidence of gallstones.

A. Aging results in decreased salivation leading to dry mouth. Aging results in increased incidence of gallstone formation. Aging results in decreased gastric acid secretion.
165. The symptoms occurring with lactose intolerance include:
A
bloating and flatulence.
B
gray stools.
C
hematemesis.
D
anorexia.

A. Lactose intolerance will produce abdominal pain, bloating, and flatulence when milk products
are consumed. Gray stools may occur with hepatitis. Hematemesis occurs with stomach or duodenal ulcers and esophageal varices. Anorexia is a loss of appetite and occurs with gastrointestinal disease, as side effect of some medications, with pregnancy, or with psychological disorders.
166. Methods to enhance abdominal wall relaxation during examination include:
A
a cool environment.
B
having the patient place arms above the head.
C
examining painful areas first.
D
positioning the patient with the knees bent.

D. Position the person supine, with the head on a pillow, the knees bent or on a pillow, and
arms at the side. Avoid having arms above the head; this increases abdominal wall tension.
Keep the room warm to avoid chilling and tensing of muscles. Painful areas should be examined
last to avoid muscle guarding.
167. In which of the following ethnic groups has the lowest incidence of osteoporosis?
A
African Americans
B
Whites
C
Asians
D
American Indians

A. African American adults have a decreased risk of fractures when compared to white adults,
and Hispanic women have a decreased risk of fractures compared to white women. The difference in fracture rates may be traced to childhood, where African American and Hispanic children have shown significantly higher bone strength than white children show. There is greater
bone density at specific bone sites in African American and Hispanic children.

Page 38 of 81

168. Heberden and Bouchard nodes are hard and nontender and are associated with:
A
osteoarthritis.
B
rheumatoid arthritis.
C
Dupuytren contracture.
D
metacarpophalangeal bursitis.

A. osteoarthritis is characterized by hard, nontender nodules, 2 to 3 mm or more. These osteophytes (bony overgrowths) of the distal interphalangeal joints are called Heberden nodes, and
those of the proximal interphalangeal joints are called Bouchard nodes. Swan-neck, boutonnire
deformity, and ulnar deviation are conditions associated with rheumatoid arthritis. Dupuytren
contracture occurs with diabetes, epilepsy, and alcoholic liver disease. Chronic hyperplasia of
the palmar fascia causes flexion contractures of the digits.Bursitis is an inflammation of the bursa.
169. The divisions of the spinal vertebrae include:
A
Cervical, thoracic, scaphoid, sacral, and clavicular.
B
Scapular, clavicular, lumbar, scaphoid, and fasciculi.
C
Cervical, thoracic, lumbar, sacral, and coccygeal.
D
Cervical, lumbar, iliac, synovial, and capsular.

C. Humans have 7 cervical, 12 thoracic, 5 lumbar, 5 sacral, and 3 to 4 coccygeal vertebrae.


170. When testing for muscle strength, the examiner should:
A
observe muscles for the degree of contraction when the individual lifts a heavy
object.
B
apply an opposing force when the individual puts a joint in flexion or extension.
C
measure the degree of force that it takes to overcome joint flexion or extension.
D
estimate the degree of flexion and extension in each joint.

B. The person should flex or extend muscle groups for each joint while the examiner applies an
opposing force. Range of motion can be described by estimating (or measuring) the degree of
flexion and extension of a joint.
.
171. The knee joint is the articulation of three bones, the:
A
femur, fibula, and patella.
B
femur, radius, and olecranon process.
C
fibula, tibia, and patella.
D
femur, tibia, and patella.

D. The knee joint is the articulation of the femur, the tibia, and the patella. The radius and ulna
are bones in the lower part of the upper extremity; the olecranon process is located on the proximal end of the ulna. The fibula is not involved in articulation of the knee joint.

Page 39 of 81
172.Crepitation is an audible sound that is produced by:
A
roughened articular surfaces moving over each other.
B
tendons or ligaments that slip over bones during motion.
C
joints that are stretched when placed in hyperflexion or hyperextension.
D
flexion and extension of an inflamed bursa.

A. Crepitation is an audible and palpable crunching or grating that accompanies movement. It


occurs when the articular surfaces in the joints are roughened. Crepitation is not the cracking
noise heard when tendons or ligaments slip over bones during motion. Hyperflexion or hyperextension is assessed with range of motion. Bursitis is an inflamed bursa. Pain may occur with
motion of the joint involved.
173. Bundles of muscle fibers that compose skeletal muscle are identified as:
A
fasciculi.
B
fasciculations.
C
ligaments.
D
tendons.
A. Each skeletal muscle is composed of bundles of muscle fibers, or fasciculi. Fasciculation is
localized uncoordinated, uncontrollable twitching of a single muscle group innervated by a single motor nerve fiber or filament. Ligaments are fibrous bands running directly from one bone to
another. Tendons are strong fibrous cords that attach skeletal muscles to bones.
174. The production of red blood cells in the bone marrow is called:
A
hematopoiesis.
B
hemolysis.
C
hemoptysis.
D
hemianopsia.

A. Hematopoiesis is the production of the red blood cells in the bone marrow (sponge-like material in the cavities of bones). Hemolysis is the breakdown of red blood cells. Hemianopsia is
blindness in half of the normal visual field. Hemoptysis is coughing up blood from the respiratory
system.
175. The musculoskeletal system functions include:
A
protection and storage.
B
movement and elimination.
C
storage and control.
D
propulsion and preservation.

A. The following are functions of the musculoskeletal system: provide support to stand erect;
allow movement; to encase and protect the inner vital organs; to produce the red blood cells in
the bone marrow; and as a reservoir for storage of essential minerals, such as calcium and
phosphorus in the bones

176. The nurse is preparing to assess a clients hip joints. Which of the following should the
nurse expect to assess with this client?

Page 40 of 81
A.
B.
C.
D.

Movement in all axes and planes


Rotation only
Flexion and extension only
Slipping and gliding motion

A. Movement in all axes and planes


Rationale: In ball and socket joints, the ball-shaped head of one bone fits into the socket of another. This joint allows movement in all axes and planes, including rotation. The shoulder and
hip joints are the only examples in the body.
177. A client postoperative hip replacement is prescribed a pillow between the legs. Which position will this pillow serve for the client?
A.
Circumduction
B.
Abduction
C.
Flexion
D.
Adduction

D. Abduction Rationale: Abduction is movement of a limb away from the midline or median
plane of the body, along the frontal plane.
178. During the assessment of a two-month-old, the nurse flexes the infants knees and compares the height of the knees. What does this technique assess?
A.
Scoliosis
B.
Head strength
C.
Allis sign
D.
Spinal bifida

C.
179. An eight-month-pregnant female client tells the nurse, Im okay except I have a backache
and I never had a backache before. Which of the following can the nurse instruct the client
about this health concern?
A.
The center of gravity has shifted putting pressure on the lower spine.
B.
The babys bottom is pressing on the spine.
C.
The babys head is pressing on the spine.
D.
Once you develop a backache in pregnancy you will always have them.

A. The center of gravity has shifted putting pressure on the lower spine.
Rationale: As the pregnancy progresses, lordosis compensates for the enlarging fetus. The center of gravity shifts forward which strains the lower spine, causing lower back pain.
180. A 47-year-old client says, I dont want to develop osteoarthritis like both of my parents
have. Which of the following can the nurse instruct this client?
A.
Calcium replacements will prevent the disorder.
B.
Taking over the counter analgesics will stop the progression of the disorder.
C.
Eating a well-balanced diet and regular exercise are the best defense.

Page 41 of 81
D.

Nothing. The disorder is genetic.

C. Eating a well-balanced diet and regular exercise are the best defense.
Rationale: A normal degenerative change associated with aging, osteoarthritis can contribute to
decreased physical activity. A well-balanced diet and regular exercise help to slow the progression of this change.
181. During the focused musculoskeletal interview, the nurse learns a client had an infected hip
joint after hip replacement surgery. What does this information suggest to the nurse?
A.
The client is at risk for future episodes of bone infections.
B.
Nothing. This is just another element of the health history.
C.
The client has osteoarthritis.
D.
The client is immunosuppressed.

A. The client is at risk for future episodes of bone infections.


Rationale: Osteomyelitis, an infection of the bone, frequently recurs in clients with a history of
previous infections.
182. A client tells the nurse about upper thigh and hip pain when standing too long. Which of
the following does this information suggest to the nurse?
A.
The client is experiencing discomfort that is consistent with her age.
B.
The client has a long-standing musculoskeletal disorder.
C.
The client is sedentary and needs to increase her endurance.
D.
The client has some degenerative disease process within the hip.

D.
183. The nurse observes a client walking incorrectly with a cane. Which of the following can the
nurse do to help this client?
A.
Place the cane in the clients other hand and observe ambulation again.
B.
Nothing. The client hasnt fallen so the method works for the client.
C.
Review the correct use of the cane with the client.
D.
Encourage the client to ambulate without the cane.

A. Place the cane in the clients other hand and observe ambulation again.
Rationale: Using a walking aid, such as a cane, can help an older client ambulate but it can also
cause falls especially if the device is not used properly.
184.During the assessment of a clients shoulder joints, the client shrugs his shoulders and then
complains of pain with abduction. What does this finding suggest to the nurse?
A.
A possible shoulder joint deformity
B.
Osteoarthritis of the joint
C.
Nothing. Continue with the assessment.
D.
Possible rotator cuff tears
D.

Page 42 of 81
185.The nurse rates a clients biceps muscle strength as a 3. What does this rating mean?
A.
The client has full range of motion without gravity.
B.
The client has full range of motion against gravity with moderate resistance.
C.
The client has full range of motion with gravity.
D.
The client has full range of motion against gravity with full resistance.

C.T he client has full range of motion with gravity.


Rationale: The rating of 3 of muscle strength is considered fair and means the client has full
range of motion with gravity.
186. A client with scoliosis complains of sharp pain in the lower back with radiation down one
leg. What should the nurse do with this information?
A.
Assess the clients knee flexion ability.
B.
Continue with the assessment, focusing on the hip joints.
C.
Stop the assessment and notify the physician of possible nerve compression.
D.
Perform the straight-leg-raising test.

D. Perform the straight-leg-raising test.


Rationale: If the client complains of sharp pain that begins in the lower back and radiates down
the leg, perform the straight-leg-raising test. The nurse should record the distribution and severity of the pain and the degree of leg elevation at the time the pain occurs. Pain with straight-legraising may indicate a herniated disk.

187. During the musculoskeletal assessment of a female client, the nurse documents that the
client has pain in the muscles and soft tissues around the head, neck, shoulders, and hips.
Which of the following disorders do these finding suggest in the client?
A.
Fibromyalgia
B.
Gout
C.
Osteoarthritis
D.
Systemic lupus erythematosus

A. Fibromyalgia
Rationale: Fibromyalgia, classified as a rheumatic disease, is characterized by pain in the muscles and soft tissues that support and surround joints. Pain is experienced in tender points of the
head, neck, shoulders, and hips.

188. The nurse observes that a 79-year-old client with a history of pulmonary tuberculosis has a
severe spinal deformity. Which of the following would most likely describe what the nurse has
assessed?
A.
Kyphosis
B.
Swan-neck deformity
C.
Lordosis
D.
Hallux Valgus

Page 43 of 81
A. Kyphosis
Rationale: Kyphosis is an exaggeration of the normal convex curve of the thoracic spine. It may
result from congenital abnormality, rheumatic conditions, compression fractures, or other disease processes including syphilis, tuberculosis, and rickets.
189. A 47-year-old client with a history of chronic back pain asks the nurse what she can do to
reduce this pain. Which of the following can the nurse instruct the client?
A.
Help to reduce the pain with weight management and a physical activity program.
B.
Talk with a pain clinic doctor about pain management.
C.
Learn to ignore the pain.
D.
Rest as much as possible.

A. Help to reduce the pain with weight management and a physical activity program.
Rationale: In a chronic back condition, education should include home and work ergonomics,
weight-reduction programs, and programs to promote physical activity, flexibility, and strength
particularly in the older adult.
190. The extrapyramidal system is located in the:
A
hypothalamus.
B
cerebellum.
C
basal ganglia.
D
medulla.

C. The basal ganglia are large bands of gray matter buried deep within the two cerebral hemispheres that form the subcortical associated motor system (the extrapyramidal system).
191. Testing the deep tendon reflexes gives the examiner information regarding the intactness
of the:
A
corticospinal tract.
B
medulla.
C
reflex arc at specific levels in the spinal cord.
D
upper motor and lower motor neuron synaptic junction.
C. Measurement of the deep tendon reflexes reveals the intactness of the reflex arc at specific
spinal levels. The medulla contains all ascending and descending fiber tracts; it has vital autonomic centers for respiration, heart, and gastrointestinal function as well as nuclei for cranial
nerves VIII through XII.
.
192. An abnormal sensation of burning or tingling is best described as:
A
paralysis.
B
paresis.
C
paresthesia.
D
paraphasia.
C. Paresthesia is an abnormal sensation such as burning or tingling. Paraphasia is a condition
in which a person hears and comprehends words but is unable to speak correctly; incoherent
words are substituted for intended words. Paralysis is a loss of motor function due to a lesion in
the neurologic or muscular system or loss of sensory innervation. Paresis is a partial or incomplete paralysis

Page 44 of 81

193. Automatic associated movements of the body are under the control and regulation of:
A
the basal ganglia.
B
the thalamus.
C
the hypothalamus.
D
Wernicke's area.

A. The basal ganglia controls automatic associated movements of the body. Wernickes area in
the temporal lobe is associated with language comprehension. The hypothalamus is a major
respiratory center with basic vital functions: temperature, appetite, sex drive, heart rate and
blood pressure control; sleep center; anterior and posterior pituitary gland regulator; and coordinator of autonomic nervous system activity and stress response. The thalamus is where sensory pathways of the spinal cord, cerebellum, and brain stem form synapses on their way to the
cerebral cortex.
194. What term is used to describe involuntary muscle movements?
A
Ataxia
B
Flaccidity
C
Athetosis
D
Vestibular function

C. Athetosis is slow, writhing, continuous, and involuntary movements of the extremities. Vestibular function is the sense of balance. Flaccidity is weak, soft, and flabby; lacking normal muscle tone. Ataxia is an impaired ability to coordinate movement, often characterized by a staggering gait and postural imbalance

195. Clonus that may be seen when testing deep tendon reflexes is characterized by a(n):
A
additional contraction of the muscle that is of greater intensity than the first.
B
set of rapid, rhythmic contractions of the same muscle.
C
parallel response in the opposite extremity.
D
contraction of the muscle that appears after the tendon is hit the second time.

B. Clonus is a set of rapid, rhythmic contractions of the same muscle.


196. The presence of primitive reflexes in a newborn infant is indicative of:
A
immaturity of the nervous system.
B
prematurity of the infant.
C
mental retardation.
D
spinal cord alterations.

A. The nervous system is not completely developed at birth, and motor activity in the newborn is
under the control of the spinal cord and medulla. The neurons are not yet myelinated. Movements are directed primarily by primitive reflexes. As the cerebral cortex develops during the
first year, it inhibits these reflexes, and they disappear at predictable times. Persistence of the
primitive reflexes is an indication of central nervous system dysfunction.

Page 45 of 81

197. Cerebellar function is tested by:


A
muscle strength assessment.
B
performance of rapid alternating movements.
C
the Phalen maneuver.
D
superficial pain and touch assessment.

B. The cerebellum controls motor coordination of voluntary movements, equilibrium, and muscle
tone. Cerebellar function is tested by balance tests (e.g., gait, Romberg test) and coordination
and skilled movements (e.g., rapid alternating movements, finger-to-finger test, finger-to-nose
test, heel-to-shin test). Muscle strength assessment examines the intactness of the motor system. The Phalen maneuver reproduces numbness and burning in a person with carpal tunnel
syndrome. Superficial pain and touch assessment examines intactness of the spinothalamic
tract
198. The _____________ coordinates movement, maintains equilibrium, and helps maintain
posture.
A
extrapyramidal system
B
cerebellum
C
upper and lower motor neurons
D
basal ganglia
B. The cerebellum controls motor coordination of voluntary movements, equilibrium (i.e., the
posture balance of the body), and muscle tone. The basal ganglia controls automatic associated
movements of the body. The extrapyramidal system maintains muscle tone and control body
movements, especially gross automatic movements such as walking.
199.During the assessment of extraocular movements, two back-and-forth oscillations of the
eyes in the extreme lateral gaze occurs. This response indicates:
A
that the individual needs to be referred for a more complete eye examination.
B
a disease of the vestibular system and that should be evaluated.
C
an expected movement of the eyes during this procedure.
D
this assessment should be repeated in 15 minutes to allow the eyes to rest.
C. Nystagmus is a back-and-forth oscillation of the eyes. End-point nystagmus, a few beats of
horizontal nystagmus at extreme lateral gaze, occurs normally.
200. The ______ reflex is an example of a _________ reflex.
A
plantar; deep tendon
B
abdominal; superficial
C
quadriceps; pathologic
D
corneal light; visceral
.
B. Superficial reflexes test the sensory receptor in the skin; the motor response is a localized
muscle contraction. Superficial reflexes include abdominal, cremasteric, and plantar (or Babinski) reflexes. The quadriceps reflex is a deep tendon reflex and is normal. The corneal light reflex assesses the parallel alignment of the eye (cranial nerves III, IV, and VI). Deep tendon reflexes test the reflex arc at the spinal level and include the biceps, triceps, brachioradialis, quadriceps, and Achilles.

Page 46 of 81

201. An elderly client is experiencing an alteration in his equilibrium and coordinated muscle
movements. The nurse realizes that these functions are controlled by which area of the nervous
system?
A.
Brain stem
B.
Cerebrum
C.
Cerebellum
D.
Diencephalon
C. Cerebellum
Rationale: The cerebellum is located below the cerebrum and behind the brain stem. It coordinates stimuli from the cerebral cortex to provide precise timing for skeletal muscle coordination
and smooth movements. The cerebellum also assists with maintaining equilibrium and muscle
tone.
202. A client with a head injury is demonstrating difficulty swallowing and talking. Which cranial
nerve might be adversely affected with this head injury?
A.
Hypoglossal
B.
Glossopharyngeal
C.
Accessory
D.
Vagus

D. Vagus
Rationale: The vagus (CN X) innervates the muscles of the throat and mouth for swallowing and
talking.
203. A client tells the nurse, I cant stand to eat my dads cooking. He seasons it way too much
for me. Which of the following could be an explanation for the nurse to provide to this client?
A.
Maybe he needs to have his vision checked.
B.
Older individuals taste level isnt as strong so they use more seasoning on food.
C.
Lets check your taste level. Maybe theres something going on with you.
D.
I think he might be dehydrated.

B. Older individuals taste level isnt as strong so they use more seasoning on food.
Rationale: The senses in older adults are not as acute as they once were. Taste is not as strong
so the older adult tends to use more seasonings on food.
204. A client with a neck injury is complaining of pain along the lateral aspect of his right hand.
Which of the following could be an explanation for this clients discomfort?
A.
He sustained an injury of the elbow.
B.
Hes experiencing pain that is reflected from the area injured in his neck.
C.
He sustained an injury of the hand that was not reported.
D.
Hes having radiating pain from his neck.

B.
205. During the focused interview of a client, the nurse wants to assess the clients memory.
Which of the following questions would be the best for the nurse to use for this purpose?

Page 47 of 81
A.
B.
C.
D.

Is one leg stronger than the other?


Do you get headaches?
What did you do today before coming here?
Have you had any changes in your ability to smell or taste?

C. What did you do today before coming here?


Rationale: Questions to assess a clients memory include Describe your memory? Do you
need to make a list or write things down so you wont forget? Do you lose things easily? and
What did you do today before you came here?

205. During the neurologic assessment of a client the nurse would like to include questions to
assess the clients ability to make healthcare decisions. Which tool can the nurse use to do this
assessment?
A.
Hopkins Competency Assessment
B.
General health questionnaire
C.
Mini-mental state examination
D.
Cornell scale

A. Hopkins Competency Assessment


Rationale: The Hopkins Competency Assessment assesses a persons ability to make decisions
about healthcare.
206. During the neurologic assessment, the nurse finds that a client is unable to calculate mathematical problems. Which of the following does this finding suggest to the nurse?
A.
The client has mental retardation.
B.
The client has schizophrenia.
C.
Nothing.
D.
The client has dementia.

C. Nothing.
Rationale: A poor response to calculations should not be considered an abnormal finding.
207. The nurse wants to assess the function of a clients accessory nerve (CN XI). Which of the
following activities can be done to assess this nerves function?
A.
Ask the client to shrug his shoulders.
B.
Ask the client to shake hands.
C.
Ask the client to stand on one foot.
D.
Ask the client to raise his head and look at the ceiling.
A. Ask the client to shrug his shoulders.
Rationale: To test the accessory nerve (CN XI), ask the client to shrug the shoulders.
208. During the assessment of sensory function of a client, the nurse learns the client is overly
sensitive to touch. Which of the following should the nurse document about this finding?
A.
Hyperesthesia
B.
Analgesia

Page 48 of 81
C.
D.

Anesthesia
Hypalgesia

A. Hyperesthesia
Rationale: Hyperesthesia is an increased sensation.
209. The nurse is unable to illicit a clients biceps reflex. Which of the following can the nurse do
to help assess this reflex?
A.
Ask the client to make a fist.
B.
Ask the client to close her eyes.
C.
Ask the client to hum.
D.
Ask the client to cross her legs.

C. Ask the client to hum.


Rationale: Before concluding that a reflex is absent or diminished, repeat the test. It may be
necessary to distract the client to achieve relaxation of the muscle. Distraction techniques include clenching the teeth, counting ceiling blocks, or humming.
210. During an assessment of an adult clients plantar reflex, the nurse notes a normal response. Which of the following would be considered normal for this client?
A.
Babinski response
B.
Inversion of the foot
C.
Dorsiflexion of the foot
D.
Plantar flexion of the foot

D. Plantar flexion of the foot


Rationale: When assessing the plantar reflex, observe for plantar flexion where the toes curl
toward the sole of the foot.
211.
Near the conclusion of the neurologic examination, the nurse notices a rapid muscle contraction
of the clients left quadriceps muscle. Which of the following could describe this assessment
finding?
A.
This is a tic.
B.
This is a tremor.
C.
This is a fasciculation.
D.
This is myoclonus.

C. This is a fasciculation.
Rationale: A fasciculation, or twitch, is an involuntary local visible muscular contraction that is
not significant if see with tired muscles.
212. A client, seen previously for herpes zoster, comes into the clinic with ongoing neurologic
changes, pain, and sensory/motor function changes. The nurse realizes this client might be experiencing:
A.
Myasthenia gravis.
B.
Meningitis.
C.
Multiple sclerosis.

Page 49 of 81
D.

Myelitis.

D. Myelitis.
Rationale: Myelitis is an inflammation of the spinal cord. Herpes zoster infection is a common
cause.
213. The nursing inservice educator is reviewing the prevalence of Alzheimers disease in the
United States. Which of the following would be an appropriate action for healthcare providers to
take regarding this disorder?
A.
Educate clients on the need for annual flu inoculations.
B.
Educate clients on the need for annual hypertension screening.
C.
Instruct clients on stress-reduction techniques.
D.
Improve the number of clients screened for the disorder.
D. Improve the number of clients screened for the disorder.
Rationale: Actions to help with the early identification and treatment of Alzheimers disease include educational programs about mental health assessments and screening/treatment programs available for the disorder.
214. The corona is:
A
a shoulder where the glans joins the shaft.
B
a hood or flap of skin over the glans.
C
a corpus spongiosum cone of erectile tissue.
D
folds of thin skin on the scrotal wall.

A. The corona is a shoulder where the glans joins the shaft.


215. Soft, pointed, fleshy papules that occur on the genitalia caused by the human papilloma
virus (HPV) are known as:
A
chancres. Syphilitic chancres are small, solitary, silvery papules that erode to a
red, round or oval, superficial ulcer with a yellowish serous discharge.
B
genital warts.
C
urethritis.
D
varicoceles.
B. Condylomata acuminata (genital warts) are soft, pointed, fleshy papules that occur on the
genitalia and are caused by the HPV. A varicocele is a dilated, tortuous varicose vein in the
spermatic cord. Urethritis is an infection of the urethra; the meatus edges are reddened, everted, and swollen.
216. Orchitis is/are:
A
a meatus opening on the dorsal side of the glans or shaft.
B
hard, subcutaneous plaques associated with painful bending of the erect penis.
C
a circumscribed collection of serous fluid in the tunica vaginalis surrounding the
testes.
D
an acute inflammation of the testes.

Page 50 of 81
D. Orchitis is an acute inflammation of the testes. A hydrocele is a circumscribed collection of
serous fluid in the tunica vaginalis surrounding the testes.
217. The ejaculatory duct is:
A
the passage formed by the joining of the vas deferens and the seminal vesicle.
B
a muscular duct continuous with the epididymis.
C
a narrow tunnel inferior to the inguinal ligament.
D
a narrow tunnel superior to the inguinal ligament.

A. The ejaculatory duct is the passage formed by the junction of the duct of the seminal vesicles
and the vas deferens through which semen enters the urethra.
218. The penis:
A
is composed of two corpora cavernosa and one corpus spongiosum.
B
a loose protective sac that is a continuation of the abdominal wall.
C
and scrotum are the internal organs of the male genital structures.
D
size is controlled by the cremaster muscle.
A. The penis is composed of three cylindrical columns of erectile tissue: two corpora cavernosa
on the dorsal side and the corpus spongiosum ventrally. The scrotum is a loose protective sac
that is a continuation of the abdominal wall. The cremaster muscle controls the size of the scrotum. The penis and scrotum are the external structures of the male genitals; the internal structures are the testis, epididymis, and the vas deferens.

219. Acute inflammation of the testes is:


A
genital herpes.
B
priapism.
C
orchitis.
D
paraphimosis.

C. Orchitis is an acute inflammation of the testes. Priapism is a prolonged painful erection of the
penis. Genital herpes is a sexually transmitted infection caused by the herpes simplex virus; the
vesicles erupt on the glans or foreskin.
220. A retention cyst in the epididymis filled with milky fluid containing sperm is called:
A
a varicocele. A varicocele is a dilated, tortuous varicose vein in the spermatic
cord.
B
a spermatocele.
C
Peyronie disease.
D
a prepuce.

B. A spermatocele is a retention cyst in the epididymis filled with milky fluid containing sperm.
Over the glans, the skin folds in and back on itself forming a hood or flap called the foreskin or
prepuce.
.

Page 51 of 81
221. The rugae:
A
is a corpus spongiosum cone of erectile tissue.
B
are folds of thin skin of the scrotal wall.
C
controls the size of the scrotum.
D
is an acute inflammation of the testes.

B. The scrotum wall consists of thin skin lying in folds, or rugae, and the underlying cremaster
muscle. The cremaster muscle controls the size of the scrotum. The penis is composed of three
cylindrical columns of erectile tissue: two corpora cavernosa on the dorsal side and the corpus
spongiosum ventrally.

222. The first sign of puberty in males is:


A
enlargement of the testes.
B
the appearance of pubic hair.
C
an increase in penis size.
D
pubic hair growth extending up the abdomen.

A. the first sign of puberty in males is the enlargement of the testes.

223. _____________ is an emergency requiring surgery.


A
A scrotal hernia
B
Epididymitis
C
Testicular torsion
D
Cryptorchidism

C. Testicular torsion is a sudden twisting of the spermatic cord; blood supply is cut off and the
testis can become gangrenous in a few hours. Emergency surgery is required.
224. The physician orders a platelet count to be performed on Mrs. Smith after breakfast. The
nurse is responsible for:
A. Instructing the patient about this diagnostic test
B. Writing the order for this test
C. Giving the patient breakfast
D. All of the above
C. A platelet count evaluates the number of platelets in the circulating blood volume. The nurse
is responsible for giving the patient breakfast at the scheduled time. The physician is responsible for instructing the patient about the test and for writing the order for the test.

225. Which of the following signs and symptoms would the nurse expect to find when assessing
an Asian patient for postoperative pain following abdominal surgery?
A. Decreased blood pressure and heart rate and shallow respirations

Page 52 of 81
B. Quiet crying
C. Immobility, diaphoresis, and avoidance of deep breathing or coughing
D. Changing position every 2 hours

C. An Asian patient is likely to hide his pain. Consequently, the nurse must observe for objective
signs. In an abdominal surgery patient, these might include immobility, diaphoresis, and avoidance of deep breathing or coughing, as well as increased heart rate, shallow respirations
(stemming from pain upon moving the diaphragm and respiratory muscles), and guarding or rigidity of the abdominal wall. Such a patient is unlikely to display emotion, such as crying.
226. High-pitched gurgles head over the right lower quadrant are:
A. A sign of increased bowel motility
B. A sign of decreased bowel motility
C. Normal bowel sounds
D. A sign of abdominal cramping
C. Hyperactive sounds indicate increased bowel motility; two or three sounds per minute indicate decreased bowel motility. Abdominal cramping with hyperactive, high pitched tinkling bowel
sounds can indicate a bowel obstruction.
227. A patient about to undergo abdominal inspection is best placed in which of the following
positions?
A. Prone
B. Trendelenburg
C. Supine
D. Side-lying

C. The supine position (also called the dorsal position), in which the patient lies on his back with
his face upward, allows for easy access to the abdomen. In the prone position, the patient lies
on his abdomen with his face turned to the side. In the Trendelenburg position, the head of the
bed is tilted downward to 30 to 40 degrees so that the upper body is lower than the legs. In the
lateral position, the patient lies on his side.
228. For a rectal examination, the patient can be directed to assume which of the following positions?
A. Genupecterol
B. Sims
C. Horizontal recumbent
D. All of the above

D. All of these positions are appropriate for a rectal examination. In the genupectoral (kneechest) position, the patient kneels and rests his chest on the table, forming a 90 degree angle
between the torso and upper legs. In Sims position, the patient lies on his left side with the left
arm behind the body and his right leg flexed. In the horizontal recumbent position, the patient
lies on his back with legs extended and hips rotated outward.
229.. During a Romberg test, the nurse asks the patient to assume which position?
A. Sitting

Page 53 of 81
B. Standing
C. Genupectoral
D. Trendelenburg

B. During a Romberg test, which evaluates for sensory or cerebellar ataxia, the patient must
stand with feet together and arms resting at the sidesfirst with eyes open, then with eyes
closed. The need to move the feet apart to maintain this stance is an abnormal finding.
230. A patient is kept off food and fluids for 10 hours before surgery. His oral temperature at 8
a.m. is 99.8 F (37.7 C) This temperature reading probably indicates:
A. Infection
B. Hypothermia
C. Anxiety
D. Dehydration

D. A slightly elevated temperature in the immediate preoperative or post operative period may
result from the lack of fluids before surgery rather than from infection. Anxiety will not cause an
elevated temperature. Hypothermia is an abnormally low body temperature.
231. All of the following can cause tachycardia except:
A. Fever
B. Exercise
C. Sympathetic nervous system stimulation
D. Parasympathetic nervous system stimulation
D. Parasympathetic nervous system stimulation of the heart decreases the heart rate as well as
the force of contraction, rate of impulse conduction and blood flow through the coronary vessels.
Fever, exercise, and sympathetic stimulation all increase the heart rate.
232. Which of the following statement is incorrect about a patient with dysphagia?
A. The patient will find pureed or soft foods, such as custards, easier to swallow than
water
B. Fowlers or semi Fowlers position reduces the risk of aspiration during swallowing
C. The patient should always feed himself
D. The nurse should perform oral hygiene before assisting with feeding.

C. A patient with dysphagia (difficulty swallowing) requires assistance with feeding. Feeding
himself is a long-range expected outcome. Soft foods, Fowlers or semi-Fowlers position, and
oral hygiene before eating should be part of the feeding regimen.

233.To assess the kidney function of a patient with an indwelling urinary (Foley) catheter, the
nurse measures his hourly urine output. She should notify the physician if the urine output is:
A. Less than 30 ml/hour
B. 64 ml in 2 hours
C. 90 ml in 3 hours
D. 125 ml in 4 hours

Page 54 of 81
A. A urine output of less than 30ml/hour indicates hypovolemia or oliguria, which is related to
kidney function and inadequate fluid intake.
234. Certain substances increase the amount of urine produced. These include:
A. Caffeine-containing drinks, such as coffee and cola.
B. Beets
C. Urinary analgesics
D. Kaolin with pectin (Kaopectate)
A. Fluids containing caffeine have a diuretic effect. Beets and urinary analgesics, such as pyridium, can color urine red. Kaopectate is an anti diarrheal medication.
235. A male patient who had surgery 2 days ago for head and neck cancer is about to make his
first attempt to ambulate outside his room. The nurse notes that he is steady on his feet and that
his vision was unaffected by the surgery. Which of the following nursing interventions would be
appropriate?
A. Encourage the patient to walk in the hall alone
B. Discourage the patient from walking in the hall for a few more days
C. Accompany the patient for his walk.
D. Consuit a physical therapist before allowing the patient to ambulate
C. A hospitalized surgical patient leaving his room for the first time fears rejection and others
staring at him, so he should not walk alone. Accompanying him will offer moral support, enabling him to face the rest of the world. Patients should begin ambulation as soon as possible
after surgery to decrease complications and to regain strength and confidence. Waiting to consult a physical therapist is unnecessary.
236. A patient has exacerbation of chronic obstructive pulmonary disease (COPD) manifested
by shortness of breath; orthopnea: thick, tenacious secretions; and a dry hacking cough. An appropriate nursing diagnosis would be:
A. Ineffective airway clearance related to thick, tenacious secretions.
B. Ineffective airway clearance related to dry, hacking cough.
C. Ineffective individual coping to COPD.
D. Pain related to immobilization of affected leg.
A.

Thick, tenacious secretions, a dry, hacking cough, orthopnea, and shortness of breath
are signs of ineffective airway clearance. Ineffective airway clearance related to dry,
hacking cough is incorrect because the cough is not the reason for the ineffective airway
clearance. Ineffective individual coping related to COPD is wrong because the etiology
for a nursing diagnosis should not be a medical diagnosis (COPD) and because no data
indicate that the patient is coping ineffectively. Pain related to immobilization of affected
leg would be an appropriate nursing diagnosis for a patient with a leg fracture.

237. Mrs. Lim begins to cry as the nurse discusses hair loss. The best response would be:
A. Dont worry. Its only temporary
B. Why are you crying? I didnt get to the bad news yet
C. Your hair is really pretty
D. I know this will be difficult for you, but your hair will grow back after the completion of
chemotheraphy

Page 55 of 81
D. I know this will be difficult acknowledges the problem and suggests a resolution to it. Dont
worry.. offers some relief but doesnt recognize the patients feelings. ..I didnt get to the bad
news yet would be inappropriate at any time. Your hair is really pretty offers no consolation or
alternatives to the patient.
238. An additional Vitamin C is required during all of the following periods except:
A. Infancy
B. Young adulthood
C. Childhood
D. Pregnancy

B. Additional Vitamin C is needed in growth periods, such as infancy and childhood, and during
pregnancy to supply demands for fetal growth and maternal tissues. Other conditions requiring
extra vitamin C include wound healing, fever, infection and stress.

239. After 1 week of hospitalization, Mr. Gray develops hypokalemia. Which of the following is
the most significant symptom of his disorder?
A. Lethargy
B. Increased pulse rate and blood pressure
C. Muscle weakness
D. Muscle irritability
C. Presenting symptoms of hypokalemia ( a serum potassium level below 3.5 mEq/liter) include
muscle weakness, chronic fatigue, and cardiac dysrhythmias. The combined effects of inadequate food intake and prolonged diarrhea can deplete the potassium stores of a patient with GI
problems.
240. Which of the following nursing interventions promotes patient safety?
A. Asses the patients ability to ambulate and transfer from a bed to a chair
B. Demonstrate the signal system to the patient
C. Check to see that the patient is wearing his identification band
D. All of the above
D. Assisting a patient with ambulation and transfer from a bed to a chair allows the nurse to
evaluate the patients ability to carry out these functions safely. Demonstrating the signal system
and providing an opportunity for a return demonstration ensures that the patient knows how to
operate the equipment and encourages him to call for assistance when needed. Checking the
patients identification band verifies the patients identity and prevents identification mistakes in
drug administration.

241. Studies have shown that about 40% of patients fall out of bed despite the use of side rails;
this has led to which of the following conclusions?
A. Side rails are ineffective
B. Side rails should not be used
C. Side rails are a deterrent that prevent a patient from falling out of bed.
D. Side rails are a reminder to a patient not to get out of bed

Page 56 of 81
D. Since about 40% of patients fall out of bed despite the use of side rails, side rails cannot be
said to prevent falls; however, they do serve as a reminder that the patient should not get out of
bed. The other answers are incorrect interpretations of the statistical data.
242. The nurses most important legal responsibility after a patients death in a hospital is:
A. Obtaining a consent of an autopsy
B. Notifying the coroner or medical examiner
C. Labeling the corpse appropriately
D. Ensuring that the attending physician issues the death certification
B. The nurse must place a pillow under the decreased persons head and shoulders to prevent
blood from settling in the face and discoloring it. She is required to bathe only soiled areas of
the body since the mortician will wash the entire body. Before wrapping the body in a shroud,
the nurse places a clean gown on the body and closes the eyes and mouth.
243. When a patient in the terminal stages of lung cancer begins to exhibit loss of consciousness, a major nursing priority is to:
A. Protect the patient from injury
B. Insert an airway
C. Elevate the head of the bed
D. Withdraw all pain medications
A. Ensuring the patients safety is the most essential action at this time. The other nursing actions may be necessary but are not a major priority.
244. The most important purpose of cleansing bed bath is:
A. To cleanse, refresh and give comfort to the client who must remain in bed
B. To expose the necessary parts of the body
C. To develop skills in bed bath
D. To check the body temperature of the client in bed

A.
245. Resonance is best describe as:
A. Sounds created by air filled lungs
B. Short, high pitch and thudding
C. Moderately loud with musical quality
D. Drum-like
A.
246. The nurse receives a report at the beginning of the shift and learns that the client scores 7
on the Glasgow Coma saclae. The nurse realizes that this client is at which of the following levels of consciousness?
A. Comatose
B. Moderate disability
C. Severe disability
D. Fully alert
247. A patient arrives at the emergency department complaining of mid-sternal chest pain.
Which of the following nursing action should take priority?

Page 57 of 81
A. A complete history with emphasis on preceding events.
B. An electrocardiogram.
C. Careful assessment of vital signs.
D. Chest exam with auscultation.
Answer: C
The priority nursing action for a patient arriving at the ED in distress is always assessment of
vital signs. This indicates the extent of physical compromise and provides a baseline by which
to plan further assessment and treatment. A thorough medical history, including onset of symptoms, will be necessary and it is likely that an electrocardiogram will be performed as well, but
these are not the first priority. Similarly, chest exam with auscultation may offer useful information after vital signs are assessed.
248. A patient has been hospitalized with pneumonia and is about to be discharged. A nurse
provides discharge instructions to a patient and his family. Which misunderstanding by the family indicates the need for more detailed information?
A. The patient may resume normal home activities as tolerated but should avoid physical
exertion and get adequate rest.
B. The patient should resume a normal diet with emphasis on nutritious, healthy foods.
C. The patient may discontinue the prescribed course of oral antibiotics once the symptoms have completely resolved.
D. The patient should continue use of the incentive spirometer to keep airways open and
free of secretions.

Answer: C
It is always critical that patients being discharged from the hospital take prescribed medications
as instructed. In the case of antibiotics, a full course must be completed even after symptoms
have resolved to prevent incomplete eradication of the organism and recurrence of infection.
The patient should resume normal activities as tolerated, as well as a nutritious diet. Continued
use of the incentive spirometer after discharge will speed recovery and improve lung function.
249. A nurse is caring for an elderly Vietnamese patient in the terminal stages of lung cancer.
Many family members are in the room around the clock performing unusual rituals and bringing
ethnic foods. Which of the following actions should the nurse take?
A. Restrict visiting hours and ask the family to limit visitors to two at a time.
B. Notify visitors with a sign on the door that the patient is limited to clear fluids only with
no solid food allowed.
C. If possible, keep the other bed in the room unassigned to provide privacy and comfort
to the family.
D. Contact the physician to report the unusual rituals and activities.

Answer: C
When a family member is dying, it is most helpful for nursing staff to provide a culturally sensitive environment to the degree possible within the hospital routine. In the Vietnamese culture, it
is important that the dying be surrounded by loved ones and not left alone. Traditional rituals
and foods are thought to ease the transition to the next life. When possible, allowing the family
privacy for this traditional behavior is best for them and the patient. Answers A, B, and D are
incorrect because they create unnecessary conflict with the patient and family.

Page 58 of 81
250. The nurse performs an initial abdominal assessment on a patient newly admitted for abdominal pain. The nurse hears what she describes as "clicks and gurgles in all four quadrants"
as well as "swishing or buzzing sound heard in one or two quadrants." Which of the following
statements is correct?
A. The frequency and intensity of bowel sounds varies depending on the phase of digestion.
B. In the presence of intestinal obstruction, bowel sounds will be louder and higher
pitched.
C. A swishing or buzzing sound may represent the turbulent blood flow of a bruit and is
not normal.
D. All of the above.
Answer: D
All of the statements are true. The gurgles and clicks described in the question represent normal
bowel sounds, which vary with the phase of digestion. Intestinal obstruction causes the sounds
to intensify as the normal flow is blocked by the obstruction. The swishing and buzzing sound of
turbulent blood flow may be heard in the abdomen in the presence of abdominal aortic aneurism, for example, and should always be considered abnormal.

251. A nurse is caring for a patient who has had hip replacement. The nurse should be most
concerned about which of the following findings?
A. Complaints of pain during repositioning.
B. Scant bloody discharge on the surgical dressing.
C. Complaints of pain following physical therapy.
D. Temperature of 101.8 F (38.7 C).

Answer: D
Post-surgical nursing assessment after hip replacement should be principally concerned with
the risk of neurovascular complications and the development of infection. A temperature of
101.8 F (38.7 C) postoperatively is higher than the low grade that is to be expected and should
raise concern. Some pain during repositioning and following physical therapy is to be expected
and can be managed with analgesics. A small amount of bloody drainage on the surgical dressing is a result of normal healing.
252. A child is admitted to the hospital with an uncontrolled seizure disorder. The admitting physician writes orders for actions to be taken in the event of a seizure. Which of the following actions would NOT be included?
A. Notify the physician.
B. Restrain the patient's limbs.
C. Position the patient on his/her side with the head flexed forward.
D. Administer rectal diazepam.

Answer: B
During a witnessed seizure, nursing actions should focus on securing the patient's safely and
curtailing the seizure. Restraining the limbs is not indicated because strong muscle contractions
could cause injury. A side-lying position with head flexed forward allows for drainage of secretions and prevents the tongue from falling back, blocking the airway. Rectal diazepam may be a
treatment ordered by the physician, who should be notified of the seizure.

Page 59 of 81

253. Emergency department triage is an important nursing function. A nurse working the evening shift is presented with four patients at the same time. Which of the following patients should
be assigned the highest priority?
A. A patient with low-grade fever, headache, and myalgias for the past 72 hours.
B. A patient who is unable to bear weight on the left foot, with swelling and bruising following a running accident.
C. A patient with abdominal and chest pain following a large, spicy meal.
D. A child with a one-inch bleeding laceration on the chin but otherwise well after falling
while jumping on his bed.

Answer: C
Emergency triage involves quick patient assessment to prioritize the need for further evaluation
and care. Patients with trauma, chest pain, respiratory distress, or acute neurological changes
are always classified number one priority. Though the patient with chest pain presented in the
question recently ate a spicy meal and may be suffering from heartburn, he also may be having
an acute myocardial infarction and require urgent attention. The patient with fever, headache
and muscle aches (classic flu symptoms) should be classified as non-urgent. The patient with
the foot injury may have sustained a sprain or fracture, and the limb should be x-rayed as soon
as is practical, but the damage is unlikely to worsen if there is a delay. The child's chin laceration may need to be sutured but is also non-urgent.
254. The nurse is preparing to assess a patient's abdomen by palpation. How should the nurse
proceed?
A. Quickly palpate a tender area to avoid any discomfort that the patient may experience.
B. Start with light palpation to detect surface characteristics and to accustom the patient
to being touched.
C. Avoid palpation of reported "tender" areas because this may cause the patient pain.
D. Begin the assessment with deep palpation, encouraging the patient to relax and take
deep breaths.

B.

Start with light palpation to detect surface characteristics and to accustom the patient to
being touched.

255. The nurse is watching a new graduate nurse perform auscultation of a patient's abdomen.
Which statement by the new graduate shows a correct understanding of the reason auscultation
precedes percussion and palpation of the abdomen?
A. "It prevents distortion of bowel sounds that might occur after percussion and palpation."
B. "It allows the patient more time to relax and therefore be more comfortable with the
physical examination."
C. "This prevents distortion of vascular sounds such as bruits and hums that might occur
after percussion and palpation."
D. "We need to determine areas of tenderness before using percussion and palpation."
A.
"It prevents distortion of bowel sounds that might occur after percussion and palpation."

Page 60 of 81

256. The nurse is watching a new graduate nurse perform auscultation of a patient's abdomen.
Which statement by the new graduate shows a correct understanding of the reason auscultation
precedes percussion and palpation of the abdomen?
A. "It prevents distortion of bowel sounds that might occur after percussion and palpation."
B. "It allows the patient more time to relax and therefore be more comfortable with the physical
examination."
C. "This prevents distortion of vascular sounds such as bruits and hums that might occur after
percussion and palpation."
D. "We need to determine areas of tenderness before using percussion and palpation."
A.

"It prevents distortion of bowel sounds that might occur after percussion and palpation."

257. A nurse notices that a patient has ascites, which indicates the presence of:
A. flatus.
B. feces.
C. fibroid tumors.
D. fluid.
D.Fluid
258. Fibrous bands running directly from one bone to another that strengthen the joint and help
prevent movement in undesirable directions are called:
A. tendons.
B. ligaments.
C. bursa.
D. cartilage.
B.

Ligaments

259. The articulation of the mandible and the temporal bone is known as the:
A. condyle of the mandible.
B. intervertebral foramen.
C. temporomandibular joint.
D. zygomatic arch of the temporal bone.
C.

temporomandibular joint

260. An 85-year-old patient comments during his annual physical that he seems to be getting
shorter as he ages. The nurse should explain that decreased height occurs with aging because:
A. there is a significant loss of subcutaneous fat.
B. there is a thickening of the intervertebral disks.
C. of the shortening of the vertebral column.
D. long bones tend to shorten with age.
C. of the shortening of the vertebral column.
261. Which of these statements concerning areas of the brain is true?

Page 61 of 81
A. Motor pathways of the spinal cord and brainstem synapse in the thalamus.
B. The hypothalamus controls temperature and regulates sleep.
C. The cerebellum is the center for speech and emotions.
D. The basal ganglia are responsible for controlling voluntary movements.

B. The hypothalamus controls temperature and regulates sleep.


262. A patient with lack of oxygen to his heart will have pain in his chest and possibly the shoulder, arms, or jaw. The nurse knows that the statement that best explains why this occurs is
which of these?
A. The lack of oxygen in his heart has resulted in decreased amount of oxygen to the
areas experiencing pain.
B. There is a lesion in the dorsal root that is preventing the sensation from being transmitted normally.
C. The sensory cortex does not have the ability to localize pain in the heart, so the pain
is felt elsewhere.
D. There is a problem with the sensory cortex and its ability to discriminate the location.
C. The sensory cortex does not have the ability to localize pain in the heart, so the pain is felt
elsewhere.
263. The nurse is checking the range of motion in a patient's knee and knows that the knee is
capable of which movement(s)?
A. flexion and extension
B. supination and pronation
C. inversion and eversion
D. circumduction
A.

Flexion and Extension

264. When performing a musculoskeletal assessment, the nurse knows that the correct approach for the examination should be:
A. anterior to posterior
B. proximal to distal
C. distal to proximal
D. posterior to anterior

B.

Proximal to distal

265. A 22-year-old man comes to the clinic for an examination after falling off his motorcycle
and landing on his left side on the handlebars. The nurse suspects that he may have injured his
spleen. Which of these statements is true regarding assessment of the spleen in this situation?
A. If an enlarged spleen is noticed, then the nurse should palpate thoroughly to determine size.
B. The spleen can be enlarged as a result of trauma.
C. An enlarged spleen should not be palpated because it can rupture easily.
D. The spleen is normally felt upon routine palpation.

Page 62 of 81
C,An enlarged spleen should not be palpated because it can rupture easily.

266. The nurse is describing a scaphoid abdomen. To the horizontal plane, a scaphoid contour of the abdomen depicts a _____ profile.
A. bulging
B. convex
C. concave
D. flat

C.Concave
267. The nurse is performing percussion during an abdominal assessment. Percussion notes
heard during the abdominal assessment may include:
A. resonance, hyperresonance, and flatness.
B. resonance, dullness, and tympany.
C. flatness, resonance, and dullness.
D. tympany, hyperresonance, and dullness.

D.Tympany, hyperresonance, and dullness.


268. During an abdominal assessment, the nurse elicits tenderness on light palpation in the
right lower quadrant. The nurse interprets that this finding could indicate a disorder of which of
these structures?
A. Sigmoid
B. Spleen
C. Gallbladder
D. Appendix

D.

Appendix

269. A 16-year-old male client tells the nurse, Im worried because I think I have misshaped
private parts. Upon examination, the nurse learns the client is concerned about his scrotum.
Which of the following can the nurse explain to this client?
A.
This is completely normal.
B.
You are right. The testicles should be even.
C.
Well, your left testicle is lower than your right.
D.
You are right. I think you should talk with the doctor.

A. This is completely normal.


Rationale: The scrotum is visibly asymmetrical with the left side extending lower than the right
because the spermatic cord is longer.

Page 63 of 81
270. A 32male-client has been diagnosed with an infection of his epididymis. Which of the following would be appropriate for the nurse to instruct this client?
A.
Its best if you wash at least three times a day.
B.
This is unusual.
C.
This is a common infection.
D.
You might not be able to have children.
C. This is a common infection.
Rationale: Epididymitis is a common infection in males
271.A 45-year-old male is concerned because of the diagnosis of indirect hernia. Which of the
following would be an appropriate response for the nurse to make to
A.
Its rare but can occur.
B.
This is the most common type of hernia.
C.
This is a medical emergency.
D.
You can have severe pain with this hernia.

B. This is the most common type of hernia


Rationale: An indirect hernia is the most common type, located within the femoral canal.
272. The mother of a two-month-old male infant says to the nurse, I think theres something
wrong. His scrotum looks so big. Which of the following can the nurse say in response to this
mother?
A.
It can appear oversized.
B.
How many diapers does he use a day?
C.
If its the size of a water balloon you should be concerned.
D.
Im sure theres nothing to worry about.

A. It can appear oversized.


Rationale: The male infants scrotum should seem oversized in comparison with the penis. If the
scrotum is enlarged and filled with fluid, a hydrocele may be present.
273. A 48-year-old male tells the nurse, I have a good wife, but Im just not interested in sex
anymore. Which of the following would be appropriate for the nurse to respond?
A. How is your appetite?
B. Are you using any recreational drugs?
C. How is your job?
D. When was the last time you took a vacation?

B. Are you using any recreational drugs?


Rationale: Sex drive can be influenced by a variety of factors including disease, medication, loss
of significant other, loss of privacy, depression, fatigue, stress, or use of alcohol or illicit drugs.
The most appropriate response would be for the nurse to ask the client about drug use.
274. The father of a 15-year-old male adolescent is concerned because he heard his son was
touching another boys private parts in the shower at school. Which of the following can the
nurse respond to this father?
A.
Has he said anything to you about it?

Page 64 of 81
B.
C.
D.

He might be homosexual.
Experimentation is common at his age and doesnt mean he will be a homosexual.
What does he do with his friends after school?

C. Experimentation is common at his age and doesnt mean he will be a homosexual.


Rationale: Homosexual experimentation is common in adolescence and may not signify homosexuality or bisexuality.
275. The nurse with a history of sexual abuse has difficulty in assessing clients sex-related organs. Which of the following can be done to support this nurses discomfort?
A. Assist this nurse to find ways to work through her own feelings.
B. Discipline the nurse for incompetent care.
C. Suggest she see a therapist.
D. Only assign the nurse to clients without these healthcare issues.
A. Assist this nurse to find ways to work through her own feelings.
Rationale: Nurses need to understand their own feelings and comfort about various aspects of
sexuality to be efficient in gathering data. It is essential for the nurse to put aside personal beliefs and values about sexual practices to be able to treat clients in a competent and nonjudgmental manner.
277. A male client tells the nurse that he is concerned because his wife hasnt been able to get
pregnant even though theyve been trying for six months. What should the nurse say in response to this client?
A.
Dont give up. It hasnt even been a year yet.
B.
Are your sexual needs being filled?
C.
How many sexual partners have you had in your lifetime?
D.
Did you have mumps as a child?

A. Dont give up. It hasnt even been a year yet.


Rationale: A couple is not considered potentially infertile unless they have been unable to conceive for a year.
278. A male client says to the nurse, Im so embarrassed. When Im erect, my penis isnt
straight. Is there some kind of surgery to fix this or something? What can the nurse respond to
this client?
A.
Surgery would make it worse.
B.
That sounds pretty bad.
C.
There is a condition where the penis is bent when erect.
D.
If you have surgery then you would be infertile.

C. There is a condition where the penis is bent when erect.


Rationale: Peyronies disease causes the shaft of the penis to be crooked during an erection.
279. In response to the nurses recommendation that he do a monthly testicular examination, a
male client says to the nurse, We dont touch our private parts where I come from. Which of
the following should the nurse do with this information?

Page 65 of 81
A.
B.
C.
D.

End the focused interview.


Document client refuses to conduct self-testicular examination.
Change the subject of conversation.
Explain the reasons why a testicular examination should be done monthly.

D. Explain the reasons why a testicular examination should be done monthly.


Rationale: Self-examination of the genitals should be performed at least monthly for early detection of changes that need follow-up.
280. The nurse is having difficulty during the focused interview of an adolescent males sexual
activity. Which of the following should the nurse do at this time?
A.
Offer information on pregnancy, birth control, and STDs.
B.
Ask the adolescents parents to talk with him.
C.
Ask if he was sexually abused as a child.
D.
Change the subject.

A. Offer information on pregnancy, birth control, and STDs.


Rationale: Whether or not a male adolescent admits to being sexually active, the nurse should
offer information on teenage pregnancy, birth control, and protection against STDs.

281.During the physical examination of a male clients scrotum, the nurse palpates a mass.
What should the nurse do next with this information?
A.
Nothing. This is a normal finding.
B.
Document mass palpated, left testicle.
C.
Perform transillumination to further assess the finding.
D.
Ask the client how long hes had a tumor in his testicle.

C. Perform transillumination to further assess the finding.


Rationale: If a mass is detected during the assessment of the scrotum, perform transillumination.

282. During the assessment of a clients prostate gland, the nurse notes multiple varicose veins
around the anal opening. Which of the following should be done with this information?
A.
Ask for assistance to complete the prostate exam.
B.
Note the characteristics of the varicosities and proceed with the exam.
C.
Ask the client to sit for a few minutes before proceeding.
D.
Stop the examination.

B. Note the characteristics of the varicosities and proceed with the exam.
Rationale: Varicosities of the hemorrhoidal veins are called hemorrhoids or piles. They are considered normal findings in adults when they are asymptomatic.
283. A 51-year-old male has never had a prostate examination. Which of the following should
be done for this client?
A.
Nothing.

Page 66 of 81
B.
C.
D.

Instruct him in the need for prostate examinations after the age of 50.
Encourage him to continue with monthly self-testicular examination.
Schedule him for this examination as soon as possible.

Instruct him in the need for prostate examinations after the age of 50.
Rationale: Advise males to follow recommendations for screening for prostate cancer annually
after 50 years of age.
284. Which of the following documentation statements indicates a normal assessment of the
perianal area?

A
"Anus moist, color darker than adjacent tissues. No lesions or discharge. Opening tightly closed."
B
"Anal area dark pink, moist, with 0.5-cm shiny blue skin sac at 5 o'clock."
C
"Anus with flabby skin sac at 7 oclock."
D
Small round opening in the anal area.

A. Normal findings during inspected of the perianal area include: anus moist and hairless, with
coarse folded skin that is more pigmented than the perianal skin; anal opening is tightly closed;
no lesions present. A small round opening in the anal area is a fistula. A shiny blue skin sac is a
thrombosed hemorrhoid. A flabby skin sac is a hemorrhoid.
285. Upon palpation the prostate gland is enlarged, nontender, firm and smooth with a palpable
central groove. This assessment finding indicates:
A
benign prostatic hypertrophy.
B
prostatitis.
C
prostate carcinoma.
D
a normal prostate gland.

A. In benign prostatic hypertrophy, the prostate gland is enlarged, nontender, firm and smooth
with a palpable central groove. In prostatitis, the prostate gland will be swollen and exquisitely
tender In prostatic carcinoma, the prostate gland will be stone-hard, irregular with fixed nodules.
A normal prostate gland does not protrude more than 1 cm into the rectum; is heart shaped,
with a palpable central groove; is smooth; is elastic, rubbery and slightly movable; and is
nontender to palpation.
286. Pruritus is the presence of:
A
an itching or burning sensation.
B
a longitudinal tear in the superficial mucosa at the anal margin.
C
blood in the stool.
D
excessive fat in the stool.

A.
287. Long-term use of laxatives frequently leads to:

Page 67 of 81
A
B
C
D

hemorrhoids.
a chronically inflamed gastrointestinal tract.
dependence.
fistula formation.

C. Long-term use of laxatives may lead to dependence. An anorectal fistula may be caused by a
chronically inflamed gastrointestinal tract. Hemorrhoids result from increased portal venous
pressure, as occurs with straining at stool, chronic constipation, pregnancy, obesity, chronic liver disease, or a low-fiber diet.
288. Toilet training (for bowel movements) in children:
A
should start after 4 years of age.
B
may begin when the nerves in the rectal area are fully myelinated.
C
can be successful as early as 12 months of age.
D
should begin after the gastrocolic reflex disappears.

B. Voluntary control of the external anal sphincter cannot occur until the nerves supplying the
area have become fully myelinated, usually around 1 to 2 years of age. The wave of peristalsis that occurs after eating is the gastrocolic reflex. This reflex is present at birth and does not
disappear.
289. Which continent has the lowest prevalence rate of prostate cancer?
A
North America
B
Australia
C
Northwestern Europe
D
Asia
D. Prostate cancer is more common in North America and northwestern Europe and is less
common in Central and South America, Africa, and Asia.
290. The etiology of a pilonidal cyst is:
A
a chronically inflamed gastrointestinal tract.
B
a tear in the superficial mucosa.
C
a congenital disorder.
D
trauma or irritant diarrheal stools.

C. A pilonidal cyst is a congenital disorder that is first diagnosed between the ages of 15 and 30
years. Fissures are tears that occur in the superficial mucosa and often result from trauma (e.g.,
passing a large, hard stool) or from irritant diarrheal stools.
.
291. An anorectal fistula is usually caused by:
A
a tear in the superficial mucosa.
B
a chronically inflamed gastrointestinal tract.
C
trauma from passing hard stools.
D
daily use of laxatives.

Page 68 of 81
B. An anorectal fistula is caused by a chronically inflamed gastrointestinal tract.
292. A patient is taking iron supplements. The patient should expect the stools to be:
A
clay colored.
B
nontarry and black.
C
tarry and black.
D
frothy.
B. Iron medication supplements will cause stools to be nontarry and black. Frothy stools have
excessive fat from a malabsorption of fat; the term to describe fatty stools is steatorrhea. Melena is the term to describe tarry, black stools; this indicates the presence of blood in the stool.
Clay colored stools are caused by the absence of bile pigment.
293. A patient tells the examiner that passing stools is painful. What term would the examiner
use to document painful bowel movements?
A
Dyschezia
B
Occult
C
Flatulence
D
Encopresis Encopresis is persistent passing of stools into clothing in a child older
than age 4 years, at which age continence would be expected.
A. Dyschezia is the painful passage of stool due to a local condition (e.g., hemorrhoids, fissure)
or constipation. Occult is a term used to describe blood in the stools that is not visible. Flatulence is the passage of gas from the anus. Encopresis is persistent passing of stools into
clothing in a child older than age 4 years, at which age continence would be expected.

294. Hematuria is a term used for:


A
bloody discharge.
B
blood in the urine.
C
bleeding after intercourse.
D
urine in the blood.

B. Blood in the urine


295. The most common sexually transmitted infection in the United States is:
A
gonorrhea.
B
syphilis.
C
chlamydia.
D
trichomoniasis.

C.
295. Cessation of menses is known as:
A
menarche.
B
menopause.
C
salpingitis.
D
adnexa.

Page 69 of 81

B. Cessation of menses is menopause. The adnexa of uterus (or uterine appendages) refers to
the structures most closely related structurally and functionally to the uterus; these structures
include the ovaries, fallopian tubes, and ligaments. Salpingitis is inflammation of the fallopian
tube. Menarche is the age of the first period.
Correct Awarded 1.0 points out of 1.0 possible points.
296. A deep recess formed by the peritoneum between the rectum and the cervix is called:
A
the Chadwick sign.
B
a cystocele.
C
a rectocele.
D
a rectouterine pouch.

D. The rectouterine pouch (or cul-de-sac of Douglas) is the deep recess between the rectum
and the cervix. A rectocele is an abnormality of the pelvic musculature where the rectum prolapses into the vagina. A cystocele is an abnormality of the pelvic musculature where the bladder prolapses into the vagina. The cervical mucosa during the second month of pregnancy is
blue which is termed the Chadwick sign. The cervix may also turn blue in any condition causing
hypoxia or venous congestion.

297. A caruncle is a(n):


A
vestibular gland located on either side of the vaginal orifice.
B
small, red mass protruding from the urethral meatus.
C
aberrant growth of endometrial tissue.
D
hard, painless nodule in the uterine wall.

B. A caruncle a small, deep red mass protruding from the urinary meatus. Endometriosis is a
disorder caused by aberrant growths of endometrial tissue scattered throughout the pelvis.
298. Adnexa is/are:
A
an absence of menstruation.
B
uterine accessory organs.
C
a membranous fold of tissue partly closing the vaginal orifice.
D
painful intercourse.

B. The adnexa of uterus (or uterine appendages) refers to the structures most closely related
structurally and functionally to the uterus; these structures include the ovaries, fallopian tubes,
and ligaments.
299. Dysmenorrhea is:
A
painful intercourse.
B
pain with defecation.
C
pain with urination.
D
pain associated with menstruation.
D.
300. Vaginal lubrication during intercourse is produced by:

Page 70 of 81
A
B
C
D

Skene glands.
Bartholin glands.
sebaceous glands.
adrenal glands.

B. The vestibular (Bartholin) glands secrete a clear lubricating mucus during intercourse.
301. The first sign of puberty in girls is:
A
the first menstrual cycle (menarche).
B
axillary hair development.
C
rapid increase in height.
D
breast and pubic hair development.

D. The first signs of puberty are breast and pubic hair development, beginning between the ages of 8 and 13 years. These signs usually occur together, but it is not abnormal if they do not
develop together. This takes about 3 years to complete.
302. Decreased estrogen levels during menopause cause:
A
an enlargement of the uterus.
B
pelvic muscles and ligaments to tighten
C
the ovaries to atrophy.
D
the cervix to enlarge and turn blue.

C. Decreased estrogen levels during menopause cause atrophy of the ovaries. Decreased estrogen levels during menopause cause the cervix to shrink and look pale with a thick, glistening
epithelium. Decreased estrogen levels during menopause cause the sacral ligaments to relax
and the pelvic musculature to weaken, which causes the uterus to drop. Decreased estrogen
levels during menopause cause the uterus to shrink related to a decrease in the myometrium.
303. When completing a health assessment of an older adult with mobility problems, the sequence should:
A
begin with the physical examination followed by the health history.
B
be from head to toe to prevent missing any important assessments.
C
be arranged to minimize the number of position changes for the patient and the
examiner.
D
start with the most invasive assessments.

C. If the older adult patient has limited mobility, the examiner should arrange the sequence to
minimize the number of position changes for the patient.
304. Which of the following assessments should be performed last on a 4-week-old infant?
A
Auscultate breath sounds
B
Otoscopic examination of the tympanic membrane
C
Weight, length, and head circumference
D
Palpate fontanels and suture lines

Page 71 of 81
B. Invasive procedures such as an otoscopic examination should be performed at the end of the
examination for an infant.
305. Which of the following tests would be used to screen a young child for developmental delays?
A
Denver II test
B
Snellen test
C
Cerebellar function test
D
Ortolani test
A. The Denver II screening test is used to screen infants and young children for developmental
delays. Cerebellar function is a test of motor coordination; rapid alternating movements is test of
cerebellar function.
306. To gain the trust of a young child, the examiner should:
A
sit down and hold the child during the examination.
B
focus on the child by asking him or her questions.
C
ask the parent to stay in the waiting area.
D
first focus on the parent as the child plays with a toy.
D. Young children may be fearful of a new person and environment; the examiner should first
focus on the parent while the child plays with a toy.
307. Which approach to a complete physical assessment should be used for an adolescent?
A
Head-to-toe approach
B
Focused approach
C
Systems approach divided into 2 or 3 visits
D
Problem-centered approach

A. A head to toe approach is appropriate for an adolescent for a complete physical examination.
308. The examiner is assessing the extraocular muscles. Which of the following tests would be
inappropriate?
A
corneal light reflex
B
six cardinal positions of gaze
C
confrontation test
D
cranial nerve III, IV, and VI testing

C. The confrontation test assesses cranial nerve II and visual fields. Extraocular muscles can be
tested by cranial nerve III, IV, and VI testing. Extraocular muscles can be tested by the six cardinal positions of gaze. Extraocular muscles can be tested by the corneal light reflex.
309. During auscultation of breath sounds, the examiner should:
A
compare sounds on the left and right side.
B
listen with the bell of the stethoscope.
C
instruct the patient to breathe in and out through the nose.
D
only listen to the posterior chest for adventitious sounds.

Page 72 of 81
A. The examiner should auscultate the lungs from side to side to compare the breath sounds.
The diaphragm of the stethoscope is used to assess lung sounds.
310. The examiner should auscultate for carotid bruits if the patient:
A
is middle-aged or older.
B
is pregnant and has gestational diabetes.
C
complains of abdominal pain.
D
has enlarged, tender cervical lymph nodes.

A. The examiner should auscultate for carotid bruits if the patient is middle-aged or older or
shows symptoms or signs of cardiovascular disease .
311.The patient sways and starts to fall when asked to stand with feet together and arms at
sides with the eyes closed. This finding would be documented as a:
A
positive Babinski sign.
B
positive Ortolani sign.
C
positive Romberg sign.
D
positive modified Allen test.
C. The Romberg test is an assessment of posture and balance (cerebellar function). Abnormal
findings occur when the person sways, falls, or widens base of feet to avoid falling A positive
Romberg sign is loss of balance that occurs when closing the eyes and occurs with cerebellar
ataxia, loss of proprioception, and loss of vestibular function. A modified Allen test is used to
evaluate the adequacy of collateral circulation prior to cannulating the radial artery. A positive
Babinski sign is an abnormal superficial reflex response.
312. The blood pressure readings for a 42-year-old male for the past two outpatient visits were
158/92 mm Hg and 146/94 mm Hg. The examiner would interpret the findings as:
A
normal.
B
prehypertension.
C
stage 1 hypertension.
D
stage 2 hypertension.

C. Stage 1 hypertension is diagnosed with SBP between 140 and 159 mm Hg or DBP between 90 and 99 mm Hg. Stage 2 hypertension is diagnosed with SBP 160 mm Hg higher or
DBP 100 mm Hg or higher. Prehypertension is diagnosed with SBP between 120 and 139 mm
Hg or DBP between 80 and 89 mm Hg.
313. The hospitalized patient does not require a full neurologic examination during every shift
assessment. Which of the following may be a way of assessing the neurologic status of the
hospitalized patient?
A
Palpate the carotid pulse. Palpating the carotid pulse is a cardiovascular assessment.
B
Offer the client a glass of water.
C
Look at the significant other throughout the examination.
D
Assign the nursing assistant to ask the patient questions and report the findings.
B. Offering the patient water is not only a courtesy but also it is an opportunity for the nurse to
note the physical data: the persons ability to hear, follow directions, cross the midline, and swal-

Page 73 of 81
low.
314. Unlicensed assistive personnel can:
A
ambulate a patient with lower extremity weakness.
B
detect adventitious breath sounds.
C
observe a change in skin lesion quality.
D
assess for electrolyte imbalances.

A. Unlicensed assistive personnel can assist a patient with activities of daily living such as ambulation. The rest are not in the scope of the unlicensed assistive personnel.
315. A 40-year-old male patient has cellulitis of the left lower extremity and no history of health
problems. Which of the following findings would need further evaluation?
A
Calluses on both hands
B
Receding hairline
C
2+ pedal pulses bilaterally
D
Irregular pulse

D. An irregular pulse is an abnormal finding and warrants further evaluation.


316. An 80-year-old patient admitted with chest pain is on a monitored unit. The hearing for this
patient should initially be assessed by:
A
normal conversation.
B
tuning fork tests.
C
the whispered voice test.
D
audiometric testing. An audiometer gives a precise quantitative measure of hearing by assessing the persons ability to hear sounds of varying frequency.

A. During the first contact with the patient (general survey or appearance), the nurse should assess the patients ability to hear a normal tone of voice. If the patient is not able to hear a normal
tone of voice, further testing may be indicated such as the whispered voice test or audiometric
testing. The tuning fork tests (Weber and Rinne) are inaccurate and should not be used for general screening. The whispered voice test is nonquantitative and documents the presence of
hearing loss but does not measure the degree of loss.
317. A hospitalized patient has pneumonia. Which of the following assessments would not be
indicated in this patient?
A
Swallowing assessment
B
Assessment of passive range of motion
C
Cardiac auscultation
D
Pain assessment

B. The nurse would not assess passive range of motion; the nurse should assess the patients
ability to turn in bed, dangle at the bedside, sit in a chair, and ambulate. In addition, the nurse
should assess the patients need for any ambulatory aids or equipment and the patients risk for
falling.

Page 74 of 81

318. What is an advantage for using SBAR during staff communication?


A
Improves verbal communication and reduces medical errors
B
Provides a complete patient health history
C
Focuses on a comprehensive physical examination
D
Avoids making recommendations SBAR communication includes

A. SBAR improves verbal communication and reduces medical errors. SBAR communication is
concise and focused; SBAR communication does not include a comprehensive physical examination does not give complete patient health history as well

319. The nurse is calling the health care provider about a patients changing condition. Which of
the following would be included in the SBAR communication?
A
Situation, Background, Assessment, and Recommendation
B
Subjective information, Background, Assessment, and Revisions needed
C
Situation, Background, All vitals, and Review of orders
D
Summary, Better plan, Accurate diagnosis, and Rights

A. SBAR communication stands for Situation, Background, Assessment, and Recommendation.

320. The nursing assistant takes the vital signs for the 12 patients on the unit. Who is responsible for interpreting the results?
A
The nursing assistant should review the results for abnormalities.
B
The registered nurse assigned to the patient(s) should interpret the vital signs.
C
The charge nurse is responsible for reviewing the vital signs on all 12 patients.
D
The unit manager must ensure that the nursing assistant reports any abnormal
results.

B. The registered nurse assigned to the patient(s) is responsible for interpreting the results. The
registered nurse is also responsible for delegating vital signs and for supervising the nursing
assistant.

321. The nurse administers an intravenous dose of pain medication. The nurse should reassess
the patient in:
A
5 minutes.
B
15 minutes.
C
30 minutes.

Page 75 of 81
D

60 minutes.

B. After an intravenous dose of pain medication, the nurse should reassess the patient in 15
minutes. If the pain medication was oral, the nurse should reassess the patient in 60 minutes.

322. Which of the following patients should the nurse assess first?
A
A 48-year-old patient with shortness of breath and pulse oximeter reading of 88%
B
A 52-year-old patient with a white blood cell count of 22,000 cells/mm3
C
A 59-year-old patient with chest pain that increases with deep inspiration
D
An 89-year-old patient with a urinary tract infection who is confused

A. The nurse should use the ABCs to determine which order to assess the patients. The nurse
should assess the 48-year-old patient with respiratory problems first (shortness of breath
and pulse oximeter reading of 88%). The 52-year-old patient with an elevated white blood
cell count should be assessed last. The 59-year-old patient with chest pain should be assessed second. The 89-year-old patient who is confused should be assessed third because
of the confusion and risk for a fall or injury.
323. Prior to the client having a gynecological examination, the client says, How can you see
my uterus this way? An appropriate response for the nurse to make would be:
A.
The fallopian tubes attach here.
B.
The uterus can be seen this way.
C.
Its to check your bladder muscle wall.
D.
This exam is to check your vagina and cervix.

D. This exam is to check your vagina and cervix.


Rationale: The vagina extends from the vestibule to the cervix at the inferior end of the uterus.
The functions of the vagina include the female organ of copulation, the birth canal, and the
channel for the exit of menstrual flow.
324. A 38-year-old female client tells the nurse, I have no interest in sexual activity since my
husband died last year. Which of the following does the nurse realize this client is demonstrating?
A.
Free from worry of unwanted pregnancy
B.
Feelings of betrayal
C.
Grief over the loss of a relationship
D.
Fear of sexual intimacy

C. Grief over the loss of a relationship


Rationale: Grief over the loss of a relationship can have long-term effects on a clients willingness to seek new relationships.
325. During the focused interview, the nurse learns that a client had the onset of menarche at
age 10. Which of the following can the nurse surmise from this information?
A.
This means the client has an endocrine malfunction.

Page 76 of 81
B.
C.
D.

This is a little early.


This is normal.
This is late.

B. This is a little early.


Rationale: Onset of menses is influenced by a variety of factors; however, it usually occurs between the ages of 11 and 14, indicating normal development. Late onset is associated with endocrine problems.
326. A client tells the nurse, I had an abortion years ago, and proceeds to cry. Which of the
following should the nurse do to support this client?
A.
Ask how the client has she been emotionally since the abortion.
B.
Change the subject.
C.
Offer a tissue.
D.
Ask if she has any other children now.

A. Ask how the client has she been emotionally since the abortion.
Rationale: Strong emotions often accompany the issue of termination of a pregnancy by either
spontaneous or surgical abortion. The nurse may want to follow up with the question How has it
been emotionally since the abortion/miscarriage?
327. A teenage client comes into the clinic with the complaint of itching and a gross thick discharge from her vaginal area. The nurse realizes this client is most likely describing:
A.
A yeast infection
B.
A syphilitic chancre sore
C.
A herpes infection
D.
Contact dermatitis

A. A yeast infection
Rationale: Yeast infections generally produce redness, pruritus (itching), and cheeselike discharge.
328.The nurse learns that a client does not perform self-examination of her genitalia. What can
the nurse do to instruct this client?
A.
Ask her why she doesnt do it.
B.
Assure her that even though its unpleasant, it needs to be done.
C.
Instruct in the ways to conduct this examination and the reasons why it should be done
monthly.
D.
Suggest she make a monthly appointment at the clinic for the nurse to examine this area.

C. Instruct in the ways to conduct this examination and the reasons why it should be done
monthly.
Rationale: Self-examination of the genitals should be performed at least monthly for early detection of changes. Teaching may be indicated if the client is not performing self-examination.

Page 77 of 81
329. A 45-year-old female tells the nurse, Confidentially, Ive never had an orgasm with a man.
Which of the following should the nurse respond to this client?
A.
So youre saying you are never sexually aroused?
B.
I dont have any suggestions for you.
C.
Im sorry to hear that.
D.
Have you talked with your partner about this?

D. Have you talked with your partner about this?


Rationale: A variety of factors may interfere with a females ability to experience orgasm. This
nurse should be nonjudgmental and attempt to assess what efforts the client has undertaken
regarding this issue.
330.During the focused interview, the nurse learns that a postmenopausal client has pain with
sex and is hot almost all of the time. Which of the following is this client describing?
A.
Symptoms of pending menstruation
B.
Sexual arousal
C.
Sexual inhibition
D.
Menopausal symptoms of dyspareunia and hot flashes

D.
331. After the examination of a client, the nurse writes labia majora symmetrical and smooth,
without lesions. Which of the following does this information imply?
A.
The client has a chancre sore on her labia majora.
B.
The clients majora are inflamed.
C.
Nothing. This is a normal finding.
D.
The client has a yeast infection.

C. Nothing. This is a normal finding.


Rationale: Normal findings include symmetrical, smooth, and lesion-free labia majora.

332. During the gynecological examination of a 19-year-old female, the nurse notes a thin layer
of skin within the vagina. This finding suggests:
A.
This client had forceful sex.
B.
Nothing. This is normal in all females.
C.
This client has never had sexual intercourse.
D.
This client has had multiple wounds to the vagina.

C.

Page 78 of 81
333. During the palpation of a clients vaginal walls, the nurse feels a bulging along the posterior
wall. This finding suggests to the nurse:
A.
Nothing. This is a normal finding.
B.
A prolapsed uterus
C.
A rectocele
D.
A cystocele

C. A rectocele
Rationale: A rectocele is a hernia that is formed when the rectum pushes into the posterior vaginal wall.
334. During the gynecological examination of a client, an erosion is seen on the surface of the
cervix. Which of the following should be done?
A.
Explain to the client that a biopsy will need to be done.
B.
Ask the client if shes ever had a surgical abortion.
C.
Ask the client if shes had traumatic intercourse.
D.
Nothing. This is a normal finding.

A. Explain to the client that a biopsy will need to be done.


Rationale: It is difficult to distinguish inflammation and erosion on the surface of the cervix without a biopsy.
335. A 65-year-old female client asks the nurse, When can I stop having these Pap smears?
An appropriate response for the nurse to make to this client would be:
A.
At least once every three years.
B.
At least once per year.
C.
After age 70 if youve had three years of normal results.
D.
Never. They are needed annually.

C. After age 70 if youve had three years of normal results.

336. A client has been diagnosed with a kidney stone, lodged within the medulla of the right kidney. Which of the following will this stone most effect?
A.
The filtration of blood
B.
The clearance of toxins
C.
The collection of urine
D.
The removal of lymph

C. The collection of urine


Rationale: The renal medulla is composed structures called pyramids and calyces. The calyces
collect urine and transport it into the renal pelvis which is the funnel-shaped superior end of the
ureter

Page 79 of 81
337. During the abdominal assessment of a male client, the nurse palpates large round mass in
the hypogastric region. Which of the following could explain what this nurse has palpated?
A.
The client is constipated.
B.
The client has kidney stones.
C.
The client has a distended or full bladder.
D.
The client has a tumor in his small intestines.

C. The client has a distended or full bladder.


Rationale: When larger amounts of urine are present, the bladder becomes distended and rises
above the symphysis pubis.
338. A client says to the nurse, I wish I could have a complete nights sleep without having to
get up every two hours to go to the bathroom. The nurse realizes this client is experiencing:
A.
Oliguria
B.
Benign prostatic hypertrophy
C.
Polyurea
D.
Nocturia

D. Nocturia
Rationale: Nocturia, nighttime urination, occurs because at rest the heart is able to pump blood
through the kidneys more efficiently facilitating the excretion of urine.
339. A client with urinary incontinence tells the nurse, I havent been to the senior center for
weeks. Which of the following would be an appropriate response for the nurse to make to this
client?
A.
Are you having problems eating?
B.
Are you getting enough rest?
C.
Can you tell me why you havent gone to the senior center?
D.
Are you having problems getting to the senior center?

C. Can you tell me why you havent gone to the senior center?
Rationale: Clients with incontinence are at increased risk for social isolation, self-esteem disturbance, and other psychosocial problems. The best response would be for the nurse to ask
the client why he hasnt gone to the center to help the client talk more about his feelings.
340. A 68-year-old female tells the nurse, Since the hysterectomy, I cant seem to hold my water. Which of the following can the nurse explain to this client?
A.
That can happen after the kind of surgery you had.
B.
Maybe the surgeon hurt your bladder with the surgery.
C.
Thats an odd complaint.
D.
Theres always adult continence pads you could use.

A. That can happen after the kind of surgery you had.

Page 80 of 81
Rationale: Urgency and stress incontinence or loss of muscle control over urination can occur
after a vaginal hysterectomy.
341. A client asks the nurse, What does having diabetes have to do with urinating? Which of
the following would be an appropriate response to make to this client?
A.
The doctors say its important.
B.
It doesnt really.
C.
Diabetes can hurt the kidneys.
D.
You are asking too many questions.

C. Diabetes can hurt the kidneys.


Rationale: Diabetes may significantly contribute to the development of renal disease.
342. A client comes into the clinic with excruciating pain in his scrotum. Which of the following
should the nurse do?
A.
Call an ambulance to take him to the emergency room.
B.
Ask the client if hes recently experienced trauma.
C.
Offer him an over the counter analgesic.
D.
Prepare to examine the kidneys.

D. Prepare to examine the kidneys.


Rationale: With renal calculi, some clients experience excruciating pain that radiates from the
flanks to the lower quadrants of the abdomen and in some cases the upper thigh and scrotum or
labium.
343. During the assessment of a clients kidneys, the nurse is unable to palpate the organs from
the back of the client. What does this finding suggest to the nurse?
A.
The clients kidneys are misshaped.
B.
Nothing. This is normal.
C.
The clients kidneys have atrophied.
D.
The clients kidneys are misplaced.

B. Nothing. This is normal.


Rationale: When enlargement occurs, the kidneys may be palpable. Otherwise they are rarely
palpable.
344.A 79-year-old client on pain medication for spinal stenosis tells the nurse, Now I cant
make it to the bathroom and Im wetting myself while I sleep. The nurse realizes this client is
describing:
A.
Functional incontinence
B.
Urge incontinence
C.
Total incontinence
D.
Reflex incontinence

A. Functional incontinence
Rationale: Functional incontinence occurs when the client is unable to reach the toilet in time
because of environmental, psychosocial, or physical factors.

Page 81 of 81

345. No breathing at all


A. orthopnea
B. bradypnea
C. tachypnea
D. eupnea
E. apnea

E. Apnea
346. Heard more on expiration loud coarse "junky sound"
A.
wheezes
B.
rales
C.
gurgles or rhonchi
D.
friction rub

C.
347. Squeaky musical sound air flowing through narrowed airway
A.
gurgles or rhochi
B.
wheezes
C.
friction rub
D.
rales

You might also like